Jump to content

Wikipedia:Reference desk/Miscellaneous

From Wikipedia, the free encyclopedia

This is an old revision of this page, as edited by The velociraptor (talk | contribs) at 02:24, 5 August 2008 (→‎Born Under Punches). The present address (URL) is a permanent link to this revision, which may differ significantly from the current revision.

Welcome to the miscellaneous section
of the Wikipedia reference desk.
Select a section:
Want a faster answer?

Main page: Help searching Wikipedia

   

How can I get my question answered?

  • Select the section of the desk that best fits the general topic of your question (see the navigation column to the right).
  • Post your question to only one section, providing a short header that gives the topic of your question.
  • Type '~~~~' (that is, four tilde characters) at the end – this signs and dates your contribution so we know who wrote what and when.
  • Don't post personal contact information – it will be removed. Any answers will be provided here.
  • Please be as specific as possible, and include all relevant context – the usefulness of answers may depend on the context.
  • Note:
    • We don't answer (and may remove) questions that require medical diagnosis or legal advice.
    • We don't answer requests for opinions, predictions or debate.
    • We don't do your homework for you, though we'll help you past the stuck point.
    • We don't conduct original research or provide a free source of ideas, but we'll help you find information you need.



How do I answer a question?

Main page: Wikipedia:Reference desk/Guidelines

  • The best answers address the question directly, and back up facts with wikilinks and links to sources. Do not edit others' comments and do not give any medical or legal advice.
See also:


July 26

Health insurance benefits in Vermont

Hi guys,does anyone know what type of job or what company in burlington vermont is known for having good benefits for their employees? like coverage of surgeries or hospital fees? and in what type of job would someone be less likely to get good benefits?. thank you! —Preceding unsigned comment added by 24.91.11.185 (talk) 03:40, 26 July 2008 (UTC)[reply]

Original research warning... IBM has decent benefits, though they just laid off 180 people, so you probably couldn't get a job there right now. Fletcher Allen Hospital probably has a decent insurance plan. They are A) a hospital and B) the second largest employer in the state after IBM. Other than those two, I'm not sure. Dismas|(talk) 11:42, 26 July 2008 (UTC)[reply]

Alexander thought himself god?

Did Alexander thought himself god?i read in a book that he believed he had a link with Greek god n Julius Caesar also believed that his family has a descended link with god which make him think that nothing can defeat him.is it true?

can u give a list of people who believed themselves god.

thank you. —Preceding unsigned comment added by 147taru (talkcontribs) 04:18, 26 July 2008 (UTC)[reply]

For your last question, see list of people who have been considered deities and self-deification. Algebraist 06:38, 26 July 2008 (UTC)[reply]

For Alexander the god, read this article, among many: [1]. And for Caesar, this one: [2]. Good luck. --Omidinist (talk) 12:23, 26 July 2008 (UTC)[reply]

Remember also that Alexander the Great lived 300+ years before Jesus, and was a polytheist. It probably wouldn't be correct to say that he thought himself to be God (i.e. the big man himself), but rather that he thought himself to be a god (that is, one among many). This probably wasn't so strange then as it sounds now, as Greek Mythology is full of instances of the gods coming down to the mortal realm and impregnating unsuspecting females. (Heracles is a good example.) The line between mortal and divine was much fuzzier in ancient times than it is in Christian philosophy. -- 128.104.112.147 (talk) 22:03, 29 July 2008 (UTC)[reply]

Police breaking down doors

In movies and TV cop shows we often see police kicking in or breaking down a locked door, to apprehend somebody inside, or to help somebody inside who's maybe tied up or beaten unconscious and can't open the door for themselves. This probably doesn't happen so often in real life, but I'm sure it happens from time to time. We never see the door being replaced, and I've often wondered who is (at least financially) responsible for fixing the door? Is this the police's responsibility? And would it depend on whether or not they had a justifiable reason for their action? Would it vary from jurisdiction to jurisdiction? Would a person whose door was kicked off its hinges by the police believing he was their suspect, but was later exonerated of any guilt, have a case for demanding the police pay for the repairs? Or even someone who was later proven to be guilty of whatever the crime was? If the police admit liability for the repairs, do they ever organise the fixing of the door themselves, or do they suggest the owner fix it and send them the bill? -- JackofOz (talk) 05:56, 26 July 2008 (UTC)[reply]

I heard on one of those "Police - Stop" kind of shows, that when the police forced entry to the wrong house the officers were joking that they would have to get the door fixed for the innocent householder. Probably not the comprehensive answer you were looking for, sorry. Astronaut (talk) 08:22, 26 July 2008 (UTC)[reply]
I doubt that someone who was later exonerated would have any increased standing for compensation for police-caused property damage if the damage was part of a legitimate investigation. If the police could convince a judge that they had a good reason to bust the door down, that there was no reasonable alternative available, then I doubt they would be liable to such property damage. In the US you probably have to file a civil suit to get the damages paid for, and then only likely in cases of mishandling or wrongdoing (e.g. a policeman destroying things as a form of intimidation, unrelated to their investigation), but I don't know that for sure. I am sure the police (in the US, anyway) don't fix it themselves or asking for a bill. Property damage in the case of certain types of investigations is inevitable and the US police only compensate if they've done something wrong. (I'm also fairly sure they won't compensate you for lost work hours that they kept you in a cell when it turns out you were innocent, for example. I know they don't do anything if, say, you lose your job while in jail, and then it turns out you are exonerated.) --98.217.8.46 (talk) 14:10, 26 July 2008 (UTC)[reply]
I don't know the answer, but something else to consider is that the owner of the door is rarely the suspect. Criminals tend to be renters (or uninvited "guests"), not homeowners, so the homeowner is the one out of luck. In many cases the homeowner or motel renter has no reason to suspect the tenant/guest is a criminal, so it doesn't seem right that they should have to foot the repair bill. StuRat (talk) 14:15, 26 July 2008 (UTC)[reply]
Yeah, I would be pretty shocked to hear that, as a rule, innocent people would have to pay for busted doors just because they were broken during a legitimate investigation. I mean, what if it's a really expensive custom-made door that costs thousands of dollars? Or what if they take apart an expensive sports car while looking for drugs? Why would the innocent citizen have to pay for what is a mistake on the police's part? I mean, that really violates my sense of justice.
Still, it seems that in at least some parts of the United States this may be the case. The first thing I came across was this post, which laments a situation exactly like this. On the other hand, that's certainly not the case in all states, as this Washington D.C. Metropolitan Police Department order makes clear. I quote: "It is the policy of this department to repair any door or other place of entry that is damaged as a result of a forcible entry by a member, or members of the department, when a member, or members may have acted upon mis­information, misinterpretation, or erroneous judgment." That's pretty straightforward.
On the other hand: "Simply because a search or seizure proves fruitless does not mean in itself that there was misinformation, misinterpretation, or erroneous judge[ment]." That sounds kinda bad, but I understand the reasoning: otherwise any drug dealer who manages to flush the drugs down the toilet in time would be off the hook. Whether someone displayed bad judgment is, of course, pretty much subject to interpretation, but I think the idea here is to say that if someone makes a mistake -- busts down the wrong door by accident or doesn't have strong evidence or valid reason to believe that forcible entry is required and justified -- then the department picks up the bill. That still leaves room for situations where the cops really do their jobs properly and act in good faith and with good reason to believe that forcible entry is required, but the suspect turns out to be innocent. However, I'm pretty sure situations like this are very rare. Which isn't much consolation to the innocent citizen, of course, but still. Anyway, I doubt the Metro PD in Washington D.C. is the only department with a comparable policy.-- Captain Disdain (talk) 16:45, 26 July 2008 (UTC)[reply]
Just because someone is innocent does not make the investigation a mistake. There are a lot of procedures and requirements for when police can break down doors (and not knock, etc.) and they have to have a legitimate reason to think that not knocking down the door would somehow really hamper things. If it turns out that they end up exonerating the suspect or whatever, that doesn't mean that making them a suspect was a mistake, or that the investigation was in error. Exoneration is as important a function of police as is conviction! --98.217.8.46 (talk) 23:31, 26 July 2008 (UTC)[reply]
Even worse than broken doors are the drug forfeiture laws in the US, which allow the police to take any car "used for criminal activity". I bet rental car companies aren't real happy about that law. StuRat (talk) 04:46, 27 July 2008 (UTC)[reply]
So the drift I'm getting is that it would be rare for the police to come to the party for the repair costs. Thank you all. I'll bear this in mind next time I get (or my door gets) busted for nefarious activities.  :) -- JackofOz (talk) 00:00, 28 July 2008 (UTC)[reply]

There is a news story about the LAPD's "Wrong Door Unit" who fix doors broken down in error available via google but unfortunately the posting of the URL here hits Wikipedia's blacklist. Google for "Repairs help rebuild LAPD's image when wrong door broken down". Nanonic (talk) 00:23, 28 July 2008 (UTC)[reply]

A friend's house door in London was broken down by the police while they were out. After they returned, and a lot of phone calls, the police conceded that they'd been given a false address by a suspect and paid for the door to be replaced. Warofdreams talk 02:00, 28 July 2008 (UTC)[reply]

Here in Quebec, a suspect shot a cop and killed him as self-defence for getting into his house brutally. He was found not guilty by the judge though he faced other charges. [3][4] 132.206.22.14 (talk) 18:39, 28 July 2008 (UTC)[reply]

Definition of city in India

What is the definition for city in India? A certain size? A charter with the word city in it? A decision of the state government? Sincerely, GeorgeLouis (talk) 06:54, 26 July 2008 (UTC)[reply]

Most probably a decision of the state government, like everywhere else. Cities and villages can fluctuate in population over time, so if city would be defined as having min. say, 50,000 inhabitants, it would be a huge hassle having a place hover between 45,000-55,000 for decades. The thing with charter is definitely not used, since any hamlet could be able to publish a charter and become a city. Admiral Norton (talk) 23:23, 26 July 2008 (UTC)[reply]
In India, it appears the term is municipal corporations and requires 200,000 population. City status is a national decision in many countries and in some, like the UK, it is down to a word in a royally granted charter so there is no reason that that couldn't be possible. In many places an exact population is required and communities gain/lose their status as population changes. See the city article. Rmhermen (talk) 22:17, 27 July 2008 (UTC)[reply]

Books are our friends and punctuality

Write a small article on Books are our friends And Also write another small article on punctuality for class 1st standard students. —Preceding unsigned comment added by 59.92.247.193 (talk) 13:15, 26 July 2008 (UTC)[reply]

No, I don't want to. You do it.
Also, what?
Poechalkdust (talk) 13:35, 26 July 2008 (UTC)[reply]
For "Books are our friends", I suggest you write about how they can keep you warm on a cold night by stoking them into the fireplace. For your article on punctuality, I suggest you take your time and get it right, even if it means handing it in a few days late. If, for some strange reason, these suggestions fail to get you the desired grade, I suggest that you do your own homework in the future. StuRat (talk) 14:07, 26 July 2008 (UTC)[reply]
"Books are our friends": Books are our friends. They don't tease us like humans do. They keep us company when humans avoid us, and even tell us stories—sometimes wonderful, sometimes tragic—to keep us from going apeshit. Screw you humans, who needs you anyway? I have books for friends!
"Punctuality": Sure, I could get to class on time, but damn it feels good to be a gangsta. − Twas Now ( talkcontribse-mail ) 22:46, 26 July 2008 (UTC)[reply]
I would have commented when the question first appeared, but I got mugged by a book and have been recuperating. OtherDave (talk) 02:35, 27 July 2008 (UTC)[reply]
Books may be you're friends, but I know they're whispering behind my back. They've stopped talking to me and the piles are beginning to scare me. They loom in a threatening manner; I don't think I should go to sleep until they leave. Why won't they leave? WHY?!

Like friends, you have to choose carefully, they can be a bad influence and some talk a lot of rot. Julia Rossi (talk) 02:22, 28 July 2008 (UTC)[reply]

Next time you're late tell your teacher you were reading. Then write an article about lying. -LambaJan (talk) 12:42, 28 July 2008 (UTC)[reply]
"Outside of a dog, a book is a man's best friend. Inside of a dog it's too dark to read." --LarryMac | Talk 13:19, 28 July 2008 (UTC)[reply]

Marshall Plan poster

Marshall Plan poster

I'm looking for a higher res version of the poster at right. Anybody have a clue where I could find such a thing? --98.217.8.46 (talk) 14:44, 26 July 2008 (UTC)[reply]

I couldn't find one online. I'd guess you'd need to find someone who actually has an original poster and get them to scan it. We do have the ship version of the poster in a slightly larger size, though: [5]. StuRat (talk) 21:39, 4 August 2008 (UTC)[reply]

MI5/6

We all know that intellegence officers/spys regularly break certain laws during the performance of their duties for example fraud, deception, forgery etc, but according to MI5/6 they do not murder individuals. My first question is does a specific public domain document list specifically what laws exactly they can and can't break if needs must or is written into the individual laws that intelligence officers can break them? Secondly can we really take their word that they don't murder people, can they really accomplish their (and the governments) goals without resorting to assasination? —Preceding unsigned comment added by 79.64.62.11 (talk) 18:23, 26 July 2008 (UTC)[reply]

If it were written into the laws then they wouldn't be breaking them. Sometimes the government will grant them exceptions under extraordinary circumstances. Other times they will just have to try and get away with it. I imagine that it is not a matter of regular policy but that rule-breaking occurs on a case-by-case basis. Plasticup T/C 23:17, 26 July 2008 (UTC)[reply]

squirt water from under toung

when i lift me toung up in my mouth water sprays out like a venum snake from under it. whot causes this? whot is it called? how many peeps have this? ~~`` —Preceding unsigned comment added by Kilop17 (talkcontribs) 18:35, 26 July 2008 (UTC)[reply]

Saliva glands do this - I occasionally happens to me, and a friend can do it at will.87.102.86.73 (talk) 18:43, 26 July 2008 (UTC)[reply]

gleeking —Preceding unsigned comment added by 66.188.139.77 (talk) 19:32, 26 July 2008 (UTC)[reply]

Fascinating - I used to be able to do this, but I had no idea it had a name, much less an article. Thanks, 66.188!— Matt Eason (Talk • Contribs) 22:13, 26 July 2008 (UTC)[reply]
I'd like to add: Wow! When I first read the question I never expected an article. Good stuff. Fribbler (talk) 22:15, 26 July 2008 (UTC)[reply]

automobile aquisition sales

I keep hearing about car dealerships having aquisition sales, how do they work and whats the catch? i find it hard to believe you can walk into a ealership and plop down $69 and walk away with a car.. is their something more to it ?Mamat1218 (talk) 18:37, 26 July 2008 (UTC)[reply]

They could have an auction where few people show up and they end up selling good cars at low prices, but they would try very hard to make sure they don't accidentally do this. Therefore, I suspect that any car sold for $69 is worth considerably less, and quite possibly has a negative value (meaning you'll have to pay more to tow it away than you can sell it for). They could also run but be disgusting, say if someone died and decomposed in it or just had a serious diarrhea attack. StuRat (talk) 01:45, 27 July 2008 (UTC)[reply]

Is this a record?

I bought a CD player in 1985. The remote still works with the original batteries. Is this a record, or is it just a CD player? —Preceding unsigned comment added by 79.76.140.78 (talk) 22:42, 26 July 2008 (UTC)[reply]

well, does it play records, or CDs? —Preceding unsigned comment added by 66.188.139.77 (talk) 23:12, 26 July 2008 (UTC)[reply]

About 15 years ago I bought a cheap Chinese watch for what is now 50 cents. It still works and I still haven't figured out how to replace the batteries. Admiral Norton (talk) 23:18, 26 July 2008 (UTC)[reply]

Yeah but can anyone beat 23 years on the same set of batteries? —Preceding unsigned comment added by 79.76.140.78 (talk) 23:48, 26 July 2008 (UTC)[reply]

I don't know about the same set of batteries, but here are some long-lasting lightbulbs. Useight (talk) 23:52, 26 July 2008 (UTC)[reply]
Yes I can believe that. Its the turning on and off that causes the thermal shock to weaken the filament. —Preceding unsigned comment added by 79.76.140.78 (talk) 23:56, 26 July 2008 (UTC)[reply]

It is literally beyond belief that the 1985 batteries still work. Perhaps someone replaced them and you were not aware of it. Edison (talk) 04:49, 27 July 2008 (UTC)[reply]

My Tandy multimeter still has the original Radio Shack AA battery it came with, when I purchased it in 1978. Obviously the current draw is very low so the battery will probably last another 30 years, provided it doesn't leak.--TrogWoolley (talk) 10:38, 27 July 2008 (UTC)[reply]
I've used the same CR-RW since 1999 every day. In 2005 I formatted it using InCD, which lets me save files to it directly like a floppy disk. I guess it must have had nearly 200 write cycles by now, so it must be near to failing. I don't use it as a main storage anymore by I regularly use it just to see how long it will last. ZigZap (talk) 15:16, 27 July 2008 (UTC)[reply]

Bum hair

Whilst looking in the mirror the other day, I noticed that I have rather a large amount of unsightly hair growing out of my anus. As a woman who likes to look her best in that area for her husband and other men friends, I would like to get it removed without visiting a clinic for waxing etc. Any advice? —Preceding unsigned comment added by 79.76.140.78 (talk) 23:16, 26 July 2008 (UTC)[reply]

Somehow I never manage to see my anus when looking in the mirror. :-) As for advice, I suggest you go to a profesional, as that's a very sensitive area and removing hair from anywhere can be painful. StuRat (talk) 01:39, 27 July 2008 (UTC)[reply]
er.... "and other men friends" ?--Shaggorama (talk) 03:21, 27 July 2008 (UTC)[reply]
Try turning the light out when you are with "other men friends," or girl friends. Then they won't notice. Edison (talk) 04:48, 27 July 2008 (UTC)[reply]
Don't feed the trolls. Malcolm XIV (talk) 10:44, 27 July 2008 (UTC)[reply]
Uh, they certainly might notice even in the dark. (As for other men friends, I realize that for a lot of people marriage equals monogamy, but that's absolutely not the standard for everyone.) -- Captain Disdain (talk) 12:16, 27 July 2008 (UTC)[reply]
I agree with Malcom XIV, so now that it's been shown to men and girls of the entire internet. Julia Rossi (talk) 02:15, 28 July 2008 (UTC)[reply]
I've heard the regrowth can be itchy. Removal of hair can also inhibit your body's natural smell reduction strategies. It might be more trouble than it's worth. Steewi (talk) 03:59, 28 July 2008 (UTC)[reply]
I can't imagine how hair can grow from a mucus membrane at all, but a troll maybe. Julia Rossi (talk) 05:05, 28 July 2008 (UTC)[reply]
To the original poster: I'm sorry you've become the butt of our jokes, but I'm afraid you left yourself wide open. StuRat (talk) 13:59, 29 July 2008 (UTC)[reply]


July 27

Phsychology

As a science, what is psychology good for?

Thanks NoEntry8 (talk) 00:07, 27 July 2008 (UTC)[reply]

"The purpose of psychology is to give us a completely different idea of the things we know best."

A quote from the French writer and philosopher Paul Valéry. —Preceding unsigned comment added by 79.76.140.78 (talk) 00:33, 27 July 2008 (UTC)[reply]

Nothing. Psychology is not a science. At best it is a social science.90.9.83.192 (talk) 12:10, 27 July 2008 (UTC)DT[reply]

Try reading the page on psychology and come back if you have more specific questions. --98.217.8.46 (talk) 13:58, 27 July 2008 (UTC)[reply]

It is a science as in study, but not a hard science (as in physics, chemistry etc). Psychology is the study of human behaviour. Julia Rossi (talk) 02:12, 28 July 2008 (UTC)[reply]
Fields (of study), arranged by purity. − Twas Now ( talkcontribse-mail ) 14:01, 28 July 2008 (UTC)[reply]

Keeping Batteries

I heard a rumour that if standard AA batteries are left in their electric appliances, such as flashlights, they will lose their power faster than if removed and left in the open. Is this true? What is the best way to store un-rechargeable batteries to maximize their shelf life? Acceptable (talk) 00:49, 27 July 2008 (UTC)[reply]

Not if the device works properly, no. It is advised to remove batteries when the device is not in use for long periods, though, since batteries can leak chemicals and damage the device. If you can't remove them, you can at least store the device with the battery compartment pointed down, so the chemicals will leak out of the device, not into the circuitry, if this happens. (Make sure the device isn't stored in a valuable piece of furniture, either.) As for long term storage of batteries, the fridge may help them last a bit longer. StuRat (talk) 01:34, 27 July 2008 (UTC)[reply]
I think you may be thinking of film, StuRat. Cold drains batteries, as any car owner in Canada could tell you. Matt Deres (talk) 03:14, 27 July 2008 (UTC)[reply]
That's a common misconception. Oil gets thick and any water in the system freezes up at low temps, which make it difficult to start the car, requiring more of a charge to turn the ignition. Thus, if you have a weak battery, the coldest days are when it will be an issue. However, as stated, it's not due to a lack of charge, but a need for additional charge. Most chemical reactions, including those that ruin film and those which discharge batteries, are slowed by lower temps. StuRat (talk) 04:20, 27 July 2008 (UTC)[reply]
I thought batteries could be revamped a tad by putting them in a warm oven. Under which conditions, I don't know and have never tried. Film in the fridge I can vote for. Julia Rossi (talk) 05:09, 30 July 2008 (UTC)[reply]
I'm a bit skeptical about that. Some batteries get a tiny bit of charge back after they are drained, just by letting them sit for a bit, whether in an oven or not, so that may have led someone to conclude that the oven did it. StuRat (talk) 13:29, 1 August 2008 (UTC)[reply]
I haven't tried an oven, but I've always understood warmth helps. When my camera batteries fail, I can always get a few more shots by removing the batteries and rolling them briskly in my hands. Warmth or friction? Gwinva (talk) 23:23, 3 August 2008 (UTC)[reply]

Reading Comic Books

I'm new to this (seeing the Dark Knight inspired me) and I was wondering how to read them. I was reading the article on Marvel Civil War and was wondering if the story would make sense if I didn't read the titles without "Civil" in them. For example, if I don't read Amazing Spider-Man #529-538 will there be a major gap in the story for me? --The Dark Side (talk) 02:18, 27 July 2008 (UTC)[reply]

No, not particularly. As someone who slogged through that mess when it was published, I can tell you it won't make any less sense if you skip those issues. Marvel has also been repeating the key issues in a series called Civil War Saga, which at least has the bonus of printing the whole thing in order. You can also check out the list of issues here, but I'd check quickly because I doubt that shoddily written list will meet notability requirements. If you don't mind some pure opinion, the Civil War thing was poorly executed and badly written. If you want a really solid story arc, try Annihilation by Marvel - awesome stuff. Or Neil Gaiman's Eternals mini series. And if you like Dark Knight, you should check out the books that inspired it: Frank Miller's seminal Batman work, and The Long Halloween, which was much more accessible. Matt Deres (talk) 03:23, 27 July 2008 (UTC)[reply]

Additionally, comics are generally designed to be reasonably accessible by new readers. Sure, there's a lot of history there, and you're going to run into references into events you haven't read about and whatnot, but generally speaking, you'll catch on easily enough. -- Captain Disdain (talk) 12:05, 27 July 2008 (UTC)[reply]
Tie-ins are any sort of work that is created to further promote or dwell deeper on core material, such as the Civil War storyline. For example, the animated film Batman: Gotham Knight, and the myriad of websites related to The Dark Knight movie are all tie-ins to the film, and they provide new content that you won't see in the movie; but, you don't have to look at those in order to understand what the movie's about. Like the Civil War tie-ins, they're just a way for people that love the core material to immerse themselves deeper in the storyline through supplemental (but not at all required) works. Kreachure (talk) 15:26, 27 July 2008 (UTC)[reply]
Oh, and also, just walking into a local comic book store and explaining that you're unfamiliar with this stuff, but would like to get into it without being confused by all of the past history will probably get you a bunch of good recommendations. -- Captain Disdain (talk) 16:10, 27 July 2008 (UTC)[reply]
I agree. I've found comic bookstore staff to be unfailingly good at recommendations, and not at all like The Simpsons' Comic Book Guy. You might also find some of the non-superhero genre graphic novels interesting. I recommend Shaun Tan and Marjane Satrapi. Within the superhero genre, the classics often cited are by Alan Moore: Watchmen, The League of Extraordinary Gentlemen and V for Vendetta (comic. Other big names are Warren Ellis, Brian K. Vaughn and Neil Gaiman. Steewi (talk) 04:05, 28 July 2008 (UTC)[reply]
I would suggest buying a graphic novel featuring the character you like instead to start with. You get a complete story arc in one book and can see whether the whole idea of comics works for you. Exxolon (talk) 00:30, 29 July 2008 (UTC)[reply]

Open Design Architecture

I am researching a building to write an article on it. A 1978 magazine describes this building as "an open design" but is there a proper name for the architecture used for the building. The building is the W. Dale Clark Library in Omaha, NE. There are a couple of photos of the building on flicker Inside 1st floor looking up Outside 14th St. View from the Lahey Mall (central park of Omaha) Inside during art project 4th floor looking down Nice or in evil (talk) 04:11, 27 July 2008 (UTC)[reply]

I'd call it open plan with a open well incorporated into the design. Roughly speaking the style is 'modern' or 'modernist'87.102.86.73 (talk) 10:29, 27 July 2008 (UTC)[reply]

Best Indian Hero Honda Bike

can anyone please tell me that which is the best indian hero honda bike? the bike should have good mileage performance and should consist four stroke engine. Please show me the poster of that bike. —Preceding unsigned comment added by 59.92.246.84 (talk) 05:42, 27 July 2008 (UTC)[reply]

I don't which is the best, and it depends on availibility in your country, but Wikipedia is also an encyclopedia which has an article on the Hero Honda. You might be able to get a decent "poster" image of a motorbike from the official site linked from the encyclopedia article. Astronaut (talk) 08:56, 27 July 2008 (UTC)[reply]

south african 2010 world cup bid

I have got an e-mail as


FROM THE DESCK OF MRS ANGELA C. ELVIS LOTTERY BID IN SOUTH AFRICA

Congratulation” you have won £800,000.00 (Eight Hundred Thousand British pounds sterling only) in the 2nd category to the draw of South African 2010 World cup bid lottery Award International programs held in Zurich, Switzerland.

Is this bid real ?


Can anyone help me on this topic. —Preceding unsigned comment added by 59.92.249.40 (talk) 05:54, 27 July 2008 (UTC)[reply]

Sorry to disappoint you but no, it's not real. It's a scam and you should delete the email. --Richardrj talk email 05:59, 27 July 2008 (UTC)[reply]
Did you buy a ticket in this lottery?. If the answer is 'no' then ask yourself why they are sending this to you. Secondly, Why are exactly the same e-mails sent to millions of other people? Thirdly, I'll bet your e-mail address on the mail is not yours. Richardrj is right, press 'delete'. Richard Avery (talk) 07:26, 27 July 2008 (UTC)[reply]
WP has an article on 419 scams, which are typically based on you transferring moneys for sundry bank fees. lawyers´expenses to the originators of the scam. Googling for Agela C. Elvis and the rest should convince you. There seem to be hundreds of such schemes, operating out of Nigeria and other locations. --Cookatoo.ergo.ZooM (talk) 10:20, 27 July 2008 (UTC)[reply]

Glad to see Elvis is alive and well and doing something productive with his--er-her time Lemon martini (talk) 14:25, 27 July 2008 (UTC)[reply]

Saying No to Court Oath

When the judicial officer reads out the oath the witness is required to say "Yes" or "I do" or something along those lines before the questioning can begin. However, what happens if a witness decides to say "No" or refuse to take the oath (say if they did not want to be involved in the trial). The first thought that jumps to mind is that they would be held in contempt of court and dealt with accordingly, but this doesn't really seem fair, because you can punish someone for lying but you can't really force them to tell the truth. Any ideas on what the outcome would be?

And I just want to make it clear that I have not done this, nor am I planning on doing it nor am I advising anyone on whether they should do it, I'm just curious about what would happen. 58.168.89.226 (talk) 08:32, 27 July 2008 (UTC)[reply]

In the UK, a witness refusing to "affirm" that he/she will tell the truth becomes known as a "hostile witness" and the jury must then make their own mind up as to the credibility of any "evidence" the witness may bring forth. 92.8.12.19 (talk) 09:39, 27 July 2008 (UTC)[reply]
Hostile witness has a different meaning in US courts. --98.217.8.46 (talk) 13:42, 27 July 2008 (UTC)[reply]
Yeah, 12.19 is wrong; Hostile witness means the same thing in the UK as in the US, and, as in the US, any witness refusing to take an oath or affirmation will get done for contempt of court. FiggyBee (talk) 06:37, 28 July 2008 (UTC)[reply]

I'm betting they'd be held in contempt of court in the US. If they're there of their own volition then they're disrupting the process of the court; if they've been subpoenaed then they are definitely in contempt of court. One way to look at the answer to this is what happened when non-Christians refused to swear on Bibles in particular (e.g. in ACLU of N.C. & Syidah Matteen v. State of North Carolina), which initially was cited as contempt of court but later got overturned as long as an appropriate alternative text was found that held the same meaning for the witness. Without getting into the entire religious question here, the original punishment—contempt of court—is probably the same as refusing to affirm in general. --98.217.8.46 (talk) 13:42, 27 July 2008 (UTC)[reply]

language

Where did the term coogans come from? They are pants with a nap at the front and cross over tapes at the back, for children. —Preceding unsigned comment added by 121.218.196.236 (talk) 10:15, 27 July 2008 (UTC)[reply]

OED, urbandictionary and wiktionary do not have this word :( --h2g2bob (talk) 15:28, 27 July 2008 (UTC)[reply]
Perhaps from child actor Jackie Coogan? Rmhermen (talk) 21:49, 27 July 2008 (UTC)[reply]
What's a nap? It sounds like you might be describing dungarees. I've never heard them called coogans, though - where are you from? It's probably a local term. --Tango (talk) 04:00, 28 July 2008 (UTC)[reply]

Solitaire card game, or "reading the cards"

I'm trying to identify what this woman is doing with playing cards. They resemble a standard deck, and the layout reminds me of a solitaire game in which a full deck of cards is laid out face up in four horizontal rows of 13,* then the aces are removed and the cards shifted to successively fill each gap with the card that follows the one to the left of the gap. (* The table in the drawing doesn't look long enough for 13, though.) My query:

  • What's the name of that game?
  • What other game might this be?
  • Alternatively, does the layout correspond to some form of fortunetelling with cards?

The drawing was made in the Theresienstadt concentration camp by Moravia-born artist Bedrich Fritta (1907-1944). -- Thanks, Deborahjay (talk) 10:34, 27 July 2008 (UTC)[reply]

I have a very similar game on my Palm PDA. Unfortunately, I don't have it with me today, but IIRC the game is called Montana Solitaire (I'll look for an online link with more info). Astronaut (talk) 15:56, 27 July 2008 (UTC)[reply]
Here's a link to an online version of the game I have. A google search reveals several sites with the same game. Also, take a look at Gaps, a similar game with slightly different rules (seems there is a whole family of "Montana type" solitaire games. Astronaut (talk) 16:06, 27 July 2008 (UTC)[reply]
Gaps is indeed the name under which I'd learned it long ago. I haven't compared the rules to "Montana Solitaire" (but am happy to have the online link :-) Would still like to know about the fortune-telling option, though. -- Deborahjay (talk) 16:57, 27 July 2008 (UTC)[reply]

Green plums

My plum tree fell down yesterday, so I've got a large box (60x30x30cm?) of unripe plums. What can I do with them?! They're unripe - plum-sized but green. --h2g2bob (talk) 14:46, 27 July 2008 (UTC)[reply]

There are some websites which claim that unripe plums can be stored at room temperature / on a soft substrate / in good light until they ripen. There is a WP article on Umeshu, which is made of unripened prunes, however, you´d have to check with the authorities in Essex. They may not like the moonshine in your garden in the middle of the day. Apparently you can also make a sort of jam out of it. --Cookatoo.ergo.ZooM (talk) 17:44, 27 July 2008 (UTC)[reply]
Cooky2, since when did moonshiners check with the authorities? Disguise it as a plum tree leaning on a tank. There's always the bath... Julia Rossi (talk) 02:08, 28 July 2008 (UTC)[reply]
Putting fruit next to (or underneath) bananas (as they ripen, or maybe even over-ripen) is supposed to speed up ripening. I'm not sure if it works with plums or if there's a limit on how unripe they can be to start with. Might be worth a try, though. --Tango (talk) 03:54, 28 July 2008 (UTC)[reply]
The ethylene gas that one ripening fruit produces can help make other fruit ripen faster, which is why one rotten apple can spoil the whole barrel, as the saying goes. From our article: "Tomatoes, bananas, and apples will ripen faster in the presence of ethylene. Bananas placed next to other fruits will produce enough ethylene to cause accelerated fruit ripening." I remember one of those "kitchen chemistry" science experiments that involved this idea. shoy (reactions) 16:54, 28 July 2008 (UTC)[reply]
We had a prune tree once. (Not a plum tree, a prune tree.) The fruit apparently had some defect where the skins weren't waterproof, causing the fruit to shrivel into prunes on the branches as the water evaporated. I wondered if this genetic mutation might have commercial value as a cheaper way to produce prunes. StuRat (talk) 14:27, 28 July 2008 (UTC)[reply]

Changing Your Name

I know that, in the U.S., you can legally change your name to pretty much anything you want. But is it possible to legally change your name so that you actually have no name? Digger3000 (talk) 16:35, 27 July 2008 (UTC)[reply]

See Name change. There are limits to what you can change your name to. Having no name seems like a legal impossibility. —D. Monack talk 17:40, 27 July 2008 (UTC)[reply]
There's Nemo for "no one" if you mean change your name to the equivalent of "no name". Julia Rossi (talk) 01:52, 28 July 2008 (UTC)[reply]

Public execution

Is there any country where people are executed on the street? —Preceding unsigned comment added by Mr.K. (talkcontribs) 17:24, 27 July 2008 (UTC)[reply]

I will try reading the articles on Saudi Arabia or Iran. —Preceding unsigned comment added by 88.6.158.156 (talk) 17:31, 27 July 2008 (UTC)[reply]

Capital punishment in Saudi Arabia, Capital punishment in Iran, Pakistan, Nigeria, North Korea - all have had recent public executions. Taliban-era Afghanistan is another fairly recent one. Rmhermen (talk) 21:46, 27 July 2008 (UTC)[reply]

Is there a way to see them online?

Is there?--96.227.17.205 (talk) 01:15, 28 July 2008 (UTC)[reply]

Try YouTube. --98.217.8.46 (talk) 02:34, 28 July 2008 (UTC)[reply]
I don't think they'd last long up on youtube. They'd probably be flagged and removed. Snuff film might be of interest to you. Steewi (talk) 04:09, 28 July 2008 (UTC)[reply]
The evening news? Julia Rossi (talk) 04:54, 28 July 2008 (UTC)[reply]
LiveLeak tends to have a lot of that kind of thing. --Sean 14:15, 28 July 2008 (UTC)[reply]

Poverty and friendship

Do poor people have more friends (and are friendly in general) than wealthy people, since they need other people? —Preceding unsigned comment added by Mr.K. (talkcontribs) 17:26, 27 July 2008 (UTC)[reply]

I think it's impossible to generalise on this.87.102.86.73 (talk) 21:49, 27 July 2008 (UTC)[reply]
Sorry for generalising, but a thought: they might be less pretentious, more accepting? Not having material wealth the poor stereotypically value relationships such as family and friendship bonds. Pride, exclusivity, snob values and status symbols are kinds of barriers the poor aren't expected to have. I like a maxim used in an art work: All we need is love/ but the poor know it's money. Julia Rossi (talk) 03:02, 28 July 2008 (UTC)[reply]
In the interests of balanced debate: You could go the other way and say wealthy people are likely to have more friends because making large amounts of money often requires good social skills. If you're impossible to get along with people aren't likely to want to do business with you. Of course, there are plenty of poor people with good social skills and plenty of rich people with bad ones, but it's possible that on average, rich people have better social skills. (Obviously, I'm ignoring inherited wealth here, but I think most wealthy people are self-made these days.) --Tango (talk) 04:15, 28 July 2008 (UTC)[reply]
No doubt about it, but I think "good social skills" is rather intra-stratum in that social skills tend to falter if not fail between social levels or groups at the personal level though the rich do seem to cross barriers to do business which is to say at the commercial level. And then, at the personal, terms of "success" vary according to one's social context and its prevailing values. Julia Rossi (talk) 04:58, 28 July 2008 (UTC)[reply]
Rich people don't have friends. They have associates posing as friends ready to stab them in the back. --mboverload@ 06:53, 28 July 2008 (UTC)[reply]
Another factor is that people's wealth tends to increase with age, while their number of friends tends to decrease with age. --Sean 14:08, 28 July 2008 (UTC)[reply]
Why do you say that most wealthy people are self-made these days? I don't have a ref but I would imagine the opposite. Zain Ebrahim (talk) 14:22, 28 July 2008 (UTC)[reply]
Because the aristocracy is pretty much a thing of the past. Wealthy people are generally earning large amounts rather than having inherited large amounts - take a look at one of the many rich lists and see how it says people made their money, very few say it was inherited. Defining "wealthy" is an arbitrary thing, but lets say it's an annual income of GBP 50,000 (US$100,000). To get that much from an inheritance you would have in inherit about £1,000,000 (assuming a 5% return, after inflation, which is probably a little optimistic). I don't have any statistics to hand, but I would expect there are far more people in jobs paying £50k than people that inherited £1m - remember, millionaires were few and far between a generation ago. --Tango (talk) 17:00, 28 July 2008 (UTC)[reply]
And remember, that £1m in cash or other paying investments - the family house doesn't count. --Tango (talk) 17:02, 28 July 2008 (UTC)[reply]
Business oligarchs make an interesting read. Bucks seem to flow upwards for some, unlike water. Julia Rossi (talk) 08:21, 29 July 2008 (UTC)[reply]
Ann Richards said of George H. W. Bush something that could be applied to many truly rich people ($100,000 / year in the U.S. is not "rich" in my opinion): "He was born on third base and thought he hit a triple." OtherDave (talk) 13:01, 29 July 2008 (UTC)[reply]

Suicide thoughts sympton or illness

Are suicide thoughts always a sympton of an illness (like depression) or can they be a illness on their own? —Preceding unsigned comment added by Mr.K. (talkcontribs) 17:28, 27 July 2008 (UTC)[reply]

Question moved to: http://en.wikipedia.org/wiki/Wikipedia:Reference_desk/Science#Suicide_thoughts_sympton_or_illness —Preceding unsigned comment added by 88.6.158.156 (talk) 17:34, 27 July 2008 (UTC)[reply]

practies paper for cat 2008

i need practies paper for cat2008 , can u help me —Preceding unsigned comment added by Anujay12 (talkcontribs) 18:38, 27 July 2008 (UTC)[reply]

There is this, but you have to sign on to download the stuff. The WP CAT article has a link to the official web site, where you can download the 2007 paper. --Cookatoo.ergo.ZooM (talk) 19:20, 27 July 2008 (UTC)[reply]


July 28

I don't know where to put this...

Can someone help me with my question here? HaGamal 09:40, 28 July 2008 (UTC)[reply]

Looks like Julia has already answered over there - but maybe they could better help you over at humanities? If you want to know how many letters are in the name, we can help. Or how the name relates to the heat death of the universe, we're all over thatOops, thought I was at SciRef :) :) Franamax (talk) 10:00, 28 July 2008 (UTC)[reply]

mh.se

where is .mh.se eg if .co.uk is the uk, .eu is the eu and .co.za is South africa, .co.sa is south america then where is .mh.se —Preceding unsigned comment added by 193.115.175.247 (talk) 11:27, 28 July 2008 (UTC)[reply]

You should probably read Top-level domain and Country code top-level domain. In short, it is the final set of characters - in your case .se which is of importance, and the answer is Sweden. --Tagishsimon (talk) 11:35, 28 July 2008 (UTC)[reply]
And www.mh.se was the domain previously held by the Mid Sweden University before they moved to their current domain. 86.21.74.40 (talk) 11:39, 28 July 2008 (UTC)[reply]
Neither South America nor the EU is a country. .sa is actually Saudi Arabia; .eu does not exist. (The EU does have a domain, but it's .eu.int.) --Anon, 15:11 UTC, July 28, 2008.
.eu does exist since 2005 /Coffeeshivers (talk) 15:44, 28 July 2008 (UTC)[reply]
Well, dang! Thanks for the correction. --Anon, 07:37 UTC, July 31.

hayward ca land and residential

I cant seem to find anything on hayward ca and the history pertainig to the land and residents of hayward ca.I live here and i would like to know about the houses and land.I want to know about the houses ive lived in.Sometimes I think its haunted and when I try to search for info I cant find anything.I type addresses and nothig.Ive typed cherryland nothing ive typed old newspaper arcticles and I have to pay please help —Preceding unsigned comment added by 67.164.85.77 (talk) 11:41, 28 July 2008 (UTC)[reply]

Our article on Hayward, California has a history section — Matt Eason (Talk • Contribs) 11:45, 28 July 2008 (UTC)[reply]

WWII

Having done some extensive research into the subject, it would seem to me that alot of survivors of the death camps claim to have had Ukrainian guards watching over them and participating in the killings. So, the article Ukrainian-German collaboration during World War II is a bit disapointing in this field. I wish to know who, what, where, when, and how ukrainians came to be guards at Auschwits and other places, they were/are not german? any info in this field would be greatly appreciated. Thanks —Preceding unsigned comment added by 193.115.175.247 (talk) 14:34, 28 July 2008 (UTC)[reply]

They were probably kapos—prisoners used to guard other prisoners, something quite common in the Nazi concentration camp system, as well as in other regimes of repression. Make one group of occupied and persecuted people watch over the others. Of course with Ukraine it is complicated, given that they had the Nazis on one side and the Soviets as the other, so how "persecuted" any given Ukrainian felt by the Germans (compared to previous persecutions by the Russians) varied quite a lot. But any of them at Auschwitz were probably there as prisoners themselves. --98.217.8.46 (talk) 14:49, 28 July 2008 (UTC)[reply]
Thabk you but no, I am not talking about Kapos, I wish to know about Ukrainian Wafen SS guards, as mentioned in books such as we wept without tears or Shoah Thanks again —Preceding unsigned comment added by 193.115.175.247 (talk) 15:22, 28 July 2008 (UTC)[reply]
This article (from www.auschwitz.org.pl) indicates there was little if any Ukrainian participation at Auschwitz. --jpgordon∇∆∇∆ 17:17, 28 July 2008 (UTC)[reply]
Follow these links to Treblinka, Sobibor, and Belzec, where you'll find particular mention of Ukrainian guards. They may have been civilian employees rather than Waffen SS personnel, if that matters. Those Nazi extermination camps are all situated on Poland's eastern border (as Auschwitz is not), near Ukraine, which may account for some of the ethnic makeup of a particular camp's guard staff. Consider, too, that there's not likely to be much in the way of testimony from camps that had so few survivors (e.g. Belzec had 2 Jews known to survive out of "at least 434,500 murdered" there). Note that Wikipedia pages on Holocaust topics have little detail (if such pages exist) compared to what you can learn from library or even Web research. -- Deborahjay (talk) 20:23, 28 July 2008 (UTC)[reply]
N.B.: Upon rereading the original query, I became aware of the phrase, "...a lot of survivors of the death camps..." -- so I focused my initial response strictly on Nazi extermination camps though by definition these had paltry few survivors. If you extend your search for testimony and documentation of Ukrainian guards at other types of Nazi concentration camps, including those described as "forced labor" camps, you may find more material. -- Deborahjay (talk) 20:33, 28 July 2008 (UTC)[reply]
The Trawniki concentration camp's training program for camp guards (to be deployed elsewhere) specifically mentions the "conscripted civilians... primarily young Ukrainians ..." to take part in Operation Reinhard (involving the three extermination camps I linked above). -- Deborahjay (talk) 20:42, 28 July 2008 (UTC)[reply]
John Demjanjuk's complicated and ongoing story comes to mind. Rmhermen (talk) 21:21, 28 July 2008 (UTC)[reply]

canoeing safety

If I am on a river Grand river Brantford Ontario and a lightening storms hits there is no place to get out where is the safest place to be? on the middle of river keep moving or close to the shore and keep moving If I can find a clearing no trees and can get out is that wise? thank you Hutch co—Preceding unsigned comment added by 99.247.207.202 (talk) 15:19, 28 July 2008 (UTC)[reply]

Get out of the boat and off of the water. I don't know of any significant stretches to the Grand that would be completely lacking in a place to bring something as small as a canoe ashore. With all the towers and tall trees, it's not particularly likely for lightning to hit you (it's not like a lake where everything is flat except for you in your boat), but it's still a needless danger. Water is also a fine conductor, so the lightning would probably not have to hit you directly to affect you. There are all kinds of houses along the river; if you don't fancy carrying your canoe back to your pickup point, some kindly soul may agree to let you sit the boat in their yard for a bit. Matt Deres (talk) 17:17, 28 July 2008 (UTC)[reply]

Architect of holy family church

i am a priest of holy family church. i am going to build a new holy family church . i want the pictures of holy family church and also the pictures of altar. i want a beautiful architecture of holy family church. —Preceding unsigned comment added by 59.92.244.252 (talk) 15:32, 28 July 2008 (UTC)[reply]

Search the web for pictures of churches and altars. Your primary requirement would seem to be an architect.--Shantavira|feed me 16:33, 28 July 2008 (UTC)[reply]
Holy Family is the name of many churches. We have article on some: Cathedral of the Holy Family, Church of the Holy Family, Holy Family Catholic Church. Rmhermen (talk) 18:19, 28 July 2008 (UTC)[reply]

Cycling

Just enjoyed the Tour de France on Eurosport. Two questions, please. Riders have earpieces (no mikes?). To what are they listening ? Most days a small group breaks away and can build a substantial lead. But it almost always is caught. Why do they do it ?86.209.155.16 (talk) 16:07, 28 July 2008 (UTC)DT[reply]

Riders' earpieces go to a team car. They tell the riders time between the groups, distance to sprints, feed zones, etc. Riders break away from the peleton on flat stages because they want to win, but they know they can't sprint. Breakaway specialists like Sylvain Chavanel and Nicolas Vogondy can keep up fast speeds to get away from the peleton, but can't sprint. Stages 18 and 19 were both won by breakaways, as were basically every mountain stage. Paragon12321 (talk) 16:23, 28 July 2008 (UTC)[reply]
The riders have microphones as well as ear pieces. The microphones are clipped to the inside of their jerseys'. That's why you sometimes see riders speaking into their jerseys. As for your second question, in addition to what Paragon12321 said, some also do it to get some TV exposure for themselves and the team (in addition to the sponsor). - Akamad (talk) 03:28, 29 July 2008 (UTC)[reply]

Many thanks90.9.214.160 (talk) 14:29, 29 July 2008 (UTC)DT[reply]

Speed dating...

I'm not sure whether to put this on the Humanities or Mathematics desk, so I'm placing it here. I know how speed dating works, but what if it's held for homosexuals? In a normal speed date, each man will only meet women and each woman will only meet men. So if we have n people (n being an even number), then the total number of meetings is (n/2)² = n²/4 (assume an arbitrary man/woman and cycle him/her through all the women/men - the other men/women will follow behind). In the homosexual case, though, we can not risk an arbitrary bipartition. So each participant will have to meet each other participant, making the total number of meetings n(n-1)/2 = n²/2 - n/2. Is something like this done in real life? How is it handled? JIP | Talk 17:28, 28 July 2008 (UTC)[reply]

I don't know how or even if such things are done in real life, but I see no reason why everyone must meet everyone else. If you want to do it that way, though, it's exactly the same as organising a round robin tournament. --Tango (talk) 17:55, 28 July 2008 (UTC)[reply]
Well, I thought, that since in conventional speed dating each man meets each woman, and vice versa, then the homosexual case should be no different - everyone should meet everyone they could potentially be interested in. Only this time it's almost twice as many meetings. JIP | Talk 18:22, 28 July 2008 (UTC)[reply]
Well, bear in mind that every participant doesn't have to meet every other participant. I mean, if you speed date 20 people, that may mean that there are 19 others that you didn't get a chance to speed date, but 20 is plenty as it is. Guaranteeing that everyone meets everyone might indeed prove to be a little problematic, but if, say, half of all the people involved meet each other, and then those halves are split again, that's going to be a lot of dating. And if a participant is left feeling that they didn't get a chance to talk to someone they really would've wanted to talk to, hey, perhaps they can muster the energy to walk up to the person and say that they'd like to grab a cup of coffee or something. I doubt this is really a problem for anyone... though admittedly the puzzle of how to conveniently organize the thing so that everyone can meet everyone is interesting. -- Captain Disdain (talk) 20:40, 28 July 2008 (UTC)[reply]
If I were pedantic I would challenge the premise we can not risk an arbitrary bipartition. If I were cynical I would point out that homosexuals in Western societies have much greater problems than the mechanics of speed dating. But I am not... wait a minute, I am both pedantic and cynical. So there you have it. Plasticup T/C 00:59, 29 July 2008 (UTC)[reply]

I think I have a solution that works if there is an odd number of tablesdoesn't work. It'd be interesting to see if there's a solution that also works for an even number of tables. --Random832 (contribs) 20:26, 29 July 2008 (UTC)[reply]

Round robin tournament#Scheduling algorithm gives a way. --Tango (talk) 01:57, 30 July 2008 (UTC)[reply]
I asked the question on the xkcd forums and basically got a solution equivalent to the same answer, so yeah. --Random832 (contribs) 03:57, 30 July 2008 (UTC)[reply]

What jobs can you do with foreign languages

I mean translator, interpreter, foreign language teacher, and what else? —Preceding unsigned comment added by Mr.K. (talkcontribs) 18:55, 28 July 2008 (UTC)[reply]

Any job which involves working with people in other countries, which includes lots of jobs in any multinational corporation. --Tango (talk) 19:04, 28 July 2008 (UTC)[reply]
Many help-desk and order-desk type jobs require or prefer bilingual staff members, especially if you speak the "other" language in a bilingual society (like Canada, for example). Many government jobs in Canada also prefer or require staff that are bilingual. Matt Deres (talk) 19:23, 28 July 2008 (UTC)[reply]
You could have interest in a career as a multi-lingual porn actor, synchronising orgasmic squirts into, say, sign language for the blind, Latin for the (oh God, here he goes again)... --Cookatoo.ergo.ZooM (talk) 19:34, 28 July 2008 (UTC)[reply]
You're going to translate to sign language for the blind? I don't know if that will work. Coolsnak3 23:44, 28 July 2008 (UTC) —Preceding unsigned comment added by Coolsnak3 (talkcontribs)
Could be as simple as a farmer needing to talk to his field hands/migrant laborers. Rmhermen (talk) 20:29, 28 July 2008 (UTC)[reply]
Perhaps some sort of work in an embassy? An ambassador? Useight (talk) 21:57, 28 July 2008 (UTC)[reply]
You usually don't start as an ambassador, but you could certainly get a job in an embassy. You could work with aid organizations; they often need translators. If your second language is Spanish you could work for most companies that operate in South America... or North America, these days. Multi-national companies are where the bi-lingual money is, and with some luck they'll pay you in Euros. Plasticup T/C 00:53, 29 July 2008 (UTC)[reply]
If you want to specifically work with the languages themselves, translation, interpreting and teaching are the main ones, yes. Consider also linguistics, which is studying the inner workings of languages. If you work with a specific language, say, Spanish, it can be combined with another interest within the community of speakers. For example, it could be combined with community work (counselling, financial advisory, new immigrant resettlement), foreign aid (development, medical care, etc.), teaching (for example, in a bilingual or international school, teaching maths or science in French), law (for multinational companies) etc. Governments often need language specialists for the current 'important' languages. In the US, currently these are Spanish, Arabic, Korean, Farsi/Dari and Chinese. These people work in Foreign Affairs, Defense, Immigration, Customs, as well as the more 'secret' departments, such as the CIA, NSA, etc. Here in Australia, there is a big need in Defense for specialists in Arabic, Chinese, Tetum and Indonesian, as well as general linguists and cryptographers. Cryptography is another area you might find interesting, especially if you also like maths and/or computer science. Multinational corporations and internet companies often need additional help in language processing (translation, culturally and linguistically adapting software, expansion into new markets, etc.). Steewi (talk) 01:16, 29 July 2008 (UTC)[reply]
Interpreter, tourism worker etc. I think most personnel on aircraft must be at leastbilingual.Avnas Ishtaroth (talk) 05:55, 29 July 2008 (UTC)[reply]
If by "personnel on aircraft" you mean flight attendants, then multiple languages is definitely a plus. For the actual flight crew, only English is required. FiggyBee (talk) 04:34, 30 July 2008 (UTC)[reply]
Yes, that is what I meant, I couldn't think of the term. ;)Avnas Ishtaroth drop me a line 01:05, 31 July 2008 (UTC)[reply]

How about foreign correspondent (journalist) ? DOR (HK) (talk) 04:22, 1 August 2008 (UTC)[reply]

main page

on average, how many people are viewing the main page per second? 79.76.186.83 (talk) 20:23, 28 July 2008 (UTC)[reply]

According to Talk:Main_Page#Results_of_main_page_traffic_experiment there were about 60 million hits over 7 days (I've no idea the source for that, though, so take it with a pinch of salt). That corresponds to an average of about 100 hits a second. --Tango (talk) 21:43, 28 July 2008 (UTC)[reply]

July 29

Canned tomato or pasta sauce

The question is very simple: Weight by weight, which one product contains more tomato? 100% pure canned/diced tomato in tomato juice or a typical mass-produced inexpensive supermarket-grade non-gourmet pasta sauce (e.g., Ragú, Hunt's)?

  • Argument 1: canned tomato > pasta sauce
    Pasta sauce contains thickening agents such as starch, sugar (high-fructose corn syrup in the U.S.), oil and water.
  • Argument 2: pasta sauce > canned tomato
    Pasta sauce is made from tomato purée or tomato paste. Even after adding water and other dirt-cheap, low-grade and barely edible ingredients, its tomato concentration is still higher than 100%.

Well, I am clueless. -- Toytoy (talk) 00:46, 29 July 2008 (UTC)[reply]

Well, it depends what you mean by "more tomato". Which contains more sea water, sea water or sea salt? 216.135.172.188 (talk) 00:50, 29 July 2008 (UTC)[reply]
Huh? Confused this one is... To OP- I think #2 is the answer. 161.222.160.8 (talk) 01:53, 29 July 2008 (UTC)[reply]
Higher than 100%? Does the Nobel Prize committee know about this? Clarityfiend (talk) 03:23, 29 July 2008 (UTC)[reply]
It sort-of makes sense if you think of a single pinch of 100% pure tomato in 1% diluted (99% water) tomato juice. Now, if you compare an average "100% pure canned tomatoes in tomato juice" with an average jar of Ragú, how many tomatoes would you need to make 100 gallons of each? --Kjoonlee 04:02, 29 July 2008 (UTC)[reply]
Assuming all the tomatoes have equal size, of course. --Kjoonlee 04:04, 29 July 2008 (UTC)[reply]
Given that a large percentage of a tomato is water, one could even ask "how much tomato is in a tomato". -- JackofOz (talk) 04:56, 29 July 2008 (UTC)[reply]

A can of thick tomato paste can easily be much higher than 100% tomato. To make 1 unit's thick tomato paste (in volume), you may need several units of fresh tomato (in volume).

Now, let's assume it takes about 1 cubic meter's fresh tomato to make 1 cubic meter's canned tomato in tomato juice. How much tomato does it take to make 1 cubic meter's typical commercial tomato-based pasta sauce? More than 1 cubic meter or less than 1 cubic meter? -- Toytoy (talk) 17:14, 29 July 2008 (UTC)[reply]

There may be one or more indicator chemicals in tomato that does not show up in other pasta paste ingridents. Maybe it's lycopene or maybe it's pectin. Does the pasta paste contain more of these chemicals than regular tomatoes? -- Toytoy (talk) 17:22, 29 July 2008 (UTC)[reply]

You're making this too complicated. Take a tomato (or a couple of tomatoes). Weigh it, cut it up and or smash it to a pulp. Air it out to dehydrate it while simultaneously dehydrating portions of the products in question whose initial weights are equal to the initial weight of the tomato. When the samples are dry, which is when they look like fruit-roll-ups (you may be able to measure moisture content with a voltage meter; the less resistance, the more water), you weigh them and then you eat them. You have no way of telling what percentage of the sauce is non-tomato material, such as onions and stuff but I'd figure it's maybe 5%-10% of the weight. The non-sauce samples may taste better with some ranch dressing or something. -LambaJan (talk) 02:38, 30 July 2008 (UTC)[reply]

Ingredients of 5 gum

Does anybody have any 5 gum on them, particularly Rain? I need to know if it contains any xylitol, and if so, how much. Thanks. --76.16.186.86 (talk) 02:03, 29 July 2008 (UTC)[reply]

I doubt you still need to know, but for the record, the ingredients list on Rain gum does not list xylitol. It is sweetened with aspartame. Matt Deres (talk) 13:02, 1 August 2008 (UTC)[reply]

Name of this muscle

What is the name of this triangular muscle on the bottom abs of men that virtually all male models seem to have? Here is a picture of the muscle: [[6]] (Note: features a man without a shirt, may not be work-safe).

And how does one develop this muscle? Acceptable (talk) 02:35, 29 July 2008 (UTC)[reply]

That line is the iliac crest and/or the inguenal ligament of his pelvis; it gets obscured pretty easily by fat and other tissue. The muscle directly above that line is the obliquus extrernus. It's difficult to build up safely, but if you have access to the right gym equipment, you can use a rack that allows you immobilize the lower half of your body and either free-bend or lift weights using your various abdominals. I've used it a number of times, but I don't know what the heck that equipment is properly called. Matt Deres (talk) 13:19, 29 July 2008 (UTC)[reply]
Just to elaborate, the reason it's difficult to build up safely is that working the abdominals often involves bending or twisting the spine while using weights, which is a risky proposition. Talk to a doctor and a trainer (in that order) before starting an intense regimen. You can mitigate the danger by building up slowly, developing all the abdominals and lower back muscles together, and making sure to never lift or bend in a sudden or jerky manner. That goes for any workout, but goes double in this case. Matt Deres (talk) 13:26, 29 July 2008 (UTC)[reply]

map shown on the flag

there are two countries shown the map on the flags of their countries. which are they? thank you124.43.48.65 (talk) 03:26, 29 July 2008 (UTC)[reply]

See Gallery of sovereign-state flags. Cyprus seems to be one... Dismas|(talk) 04:36, 29 July 2008 (UTC)[reply]
The flag of Kosovo also includes a map. Whether or not it's a country is a matter of some controversy. —D. Monack talk 06:38, 29 July 2008 (UTC)[reply]
The flag of Tuvalu arguably is also a map. —D. Monack talk 06:40, 29 July 2008 (UTC)[reply]
Similarly, the flag of Nauru shows the island's position relative to the Equator. There's also the flag of Bumbunga, though I doubt Bumbunga would be counted as a country! AJHW (talk) 13:54, 29 July 2008 (UTC)[reply]
The only countries currently with "real" maps on their flags are Cyprus and Kosovo. Others have stylised represenatations - these include Bosnia and Herzegovina, Tuvalu, and Nauru. Cambodia briefly had a map on its flag (which was vaguely similar to the current Kosovo flag), and a proposed flag of Tokelau which narrowly missed being adopted had stars as islands, as per Tuvalu. Grutness...wha? 23:39, 29 July 2008 (UTC)[reply]

distance

what is the exact distance of doyong in calasiao to downtown dagupan

Dagupan and Calasiao are apparently both in the Philippines, and Doyong is apparently one of the barangays in Calasiao. What kind of distance do you want ? Driving distance, great circle distance (shortest route on the Earth's surface), or straight-line distance (shortest distance through the Earth) ? I can only come up with a very rough approximation of 3 miles. StuRat (talk) 13:23, 29 July 2008 (UTC)[reply]
I've spent some time in the Philippines, so I'd estimate that it would take you between 30 minutes and an hour to travel on a bus between the two locations. You'll have to wait for the bus to fill up and then it'll drive really slowly. Useight (talk) 14:52, 29 July 2008 (UTC)[reply]
That's 3-6 miles per hour. You could walk at 3 mph. StuRat (talk) 00:09, 31 July 2008 (UTC)[reply]

Sort of water filter system in US

Can I know what sort of water filter system is most common in US?

Is is Reverse OSmosis, UV, or other sort?

Thanks!

Although we don't think of them as water filter systems, I would think a water softener might be the most common whole-house filter. They replace minerals in hard water with sodium, so do filter out those minerals. Filters at the sink are usually reverse osmosis and/or activated charcoal. Then there are the pitchers that filter water, which are always activated charcoal, as far as I know. StuRat (talk) 13:09, 29 July 2008 (UTC)[reply]

Size of Virus and heavy metal

Can anybody show me the picture or scale of virus and heavy metal which exist in our drinking water?

Which sort of filter can actually remove this kind of creatures and contaminant? Thanks!!

A virus is going to be many orders of magnitude larger than a single molecule of any heavy metal. A reverse osmosis filter should remove the viruses and most of the heavy metals. An activated charcoal filter is likely less effective. Note that many reverse osmosis filters have a defective design, in that they lack the cycle which purges the filter of accumulated debris, and this clogs them on a regular basis. Be sure to get one with a purge cycle (or that allows you to manually purge it, at the very least). This doesn't apply to activated charcoal filters, as they must be periodically replaced (and thus usually end up costing more in the long run). StuRat (talk) 13:02, 29 July 2008 (UTC)[reply]

Networking technology in restaurants

What kind of fancy technology do they have in restaurants now? I know people can drag their laptops into Starbucks and get free wireless 'net, but what other stuff is there in terms of networking? Also, what's going to happen in regards to Microsoft Surface?Avnas Ishtaroth (talk) 05:53, 29 July 2008 (UTC)[reply]

Modifying signature

How does one go about putting fancy fonts, colours etc. into their signature? Is it HTML, BBCode or some Wikimarkup? Does anyone have a list of functions, thanks.Avnas Ishtaroth (talk) 05:56, 29 July 2008 (UTC)[reply]

While I'm at it, how do I check exactly how many edits I've got?Avnas Ishtaroth (talk) 06:07, 29 July 2008 (UTC)[reply]
It's just HTML. You do have to check the "Raw signature" box though once you modify it. As for the second bit, there was a tool that someone created to check your edit count with but I've forgotten whose it was. It wasn't on site though. Dismas|(talk) 06:24, 29 July 2008 (UTC)[reply]
Edit counter. --CWY2190(talkcontributions) 06:56, 29 July 2008 (UTC)[reply]
You don't even need that nowadays, it's in your "my preferences" already, where you can also change your signature. --antilivedT | C | G 07:55, 29 July 2008 (UTC)[reply]

Thankyou! I'm happy now. Avnas Ishtaroth drop me a line 11:24, 29 July 2008 (UTC)[reply]

The "My Preferences" count includes deleted edits while the wannabe_kate tool does not. Just for clarification. Useight (talk) 14:49, 29 July 2008 (UTC)[reply]
Cool, I didn't realise that! the wub "?!" 08:31, 31 July 2008 (UTC)[reply]

Bigger than Jesus?

Can anyone tell me Jesus' adult height and weight, I want to be able to claim that I'm bigger than him.... failing that where can I find data on the average height and weight of middle eastern males in that period so I can at least say that I'm more than likely bigger than jesus (was)?

Here[7] are some figures based on Jewish males as well as portraits. Pity is he's got the age advantage according to some. :) Julia Rossi (talk) 08:16, 29 July 2008 (UTC)[reply]
But are you bigger than the Beatles? -LambaJan (talk) 03:00, 30 July 2008 (UTC)[reply]
Depending on one's opinion of the authenticity of the Shroud of Turin, the characteristics of the shroud image could be used to approximate height and weight. I seem to recall reading such an estimate in a book on the Shroud but can't think of what the title was. User:Jwrosenzweig editing as 71.112.34.57 (talk) 09:42, 4 August 2008 (UTC)[reply]

map

where can i find the map of small countries in a reasonable large size? thank you.124.43.65.210 (talk) 08:13, 29 July 2008 (UTC)[reply]

To see the small countries you need to have a map of each of them alone, unless they are right next to each other, then you can show a group. For example, on any map that shows Russia, the Baltic States will seem tiny by comparison. However, just go to Google + Maps and type in "Latvia" for Latvia alone (you can then zoom out to see all the Baltic nations at once). You can make each map bigger by picking on the "<" symbol on the left side of the map, and by increasing screen resolution to the max on the Windows O/S by using Start (+ Settings) + Control Panel + Display Icon + Settings Tab to slide the resolution slider all the way to the right. StuRat (talk) 12:30, 29 July 2008 (UTC)[reply]
If you don't like Google's own maps, you can go to Google + Images and type in "Latvia Map" for a list of all Latvia maps found on the Internet. Select "Extra Large Images" from the pull-down menu to restrict the results. Pick on the map you want, to go to it's site. I found this nice zoomable map that way for Latvia, for example: [8]. StuRat (talk) 12:42, 29 July 2008 (UTC)[reply]

Is Obama black?

Is he? I would call him mulatto and it is not meant as an insult, just fact.

"Black" and "White" and social constructs anyway. His father was black Kenyan and his mother was a white woman from Kansas. He is of mixed decent - call him what you will. Oh, and mulatto is not PC these days. There is some other euphemism now. Plasticup T/C 11:03, 29 July 2008 (UTC)[reply]
Yes, the proper PC terminology is to say "he's got a bit of cream in his coffee". :-) StuRat (talk) 12:51, 29 July 2008 (UTC)[reply]
Hahahahaha, I'm still laughing about this one. —Keenan Pepper 13:23, 29 July 2008 (UTC)[reply]
The fact is that almost all African-Americans are various mixtures, modern and historical, between African and European populations (along with Native Americans, while we're at it). Historically any child of any sort of mixture of Black and non-Black populations is usually considered Black, no matter how small or large the percentage is, but this just makes the fact that it has little to do with biology all the clearer. --98.217.8.46 (talk) 14:13, 29 July 2008 (UTC)[reply]
Agreed, it's more of a sociological than a (scientific) "fact" question. See also the article on one-drop rule for some historical perspective and more contemporary dynamics. ---Sluzzelin talk 15:37, 29 July 2008 (UTC)[reply]
I'd say the current preferred term is biracial or multiracial. Marco polo (talk) 01:51, 30 July 2008 (UTC)[reply]
The relevant article is hypodescent. The important aspect to this is how he's perceived in the world and the issues he has to deal with. He may be mixed (or however you want to call it) at home, and he may even identify himself that way on documents and other parts of his life, but the social pressures are real and he has to deal with the reality of being a black man in America as much as any other black man does. -LambaJan (talk) 02:58, 30 July 2008 (UTC)[reply]
According to the pencil test, he is coloured or multiracial. Zain Ebrahim (talk) 10:06, 4 August 2008 (UTC)[reply]

Looking cool with scary eyes!

(Request for medical advice removed. Note that saying "this is not a medical question" does not mean that this is not a medical question.) - EronTalk 14:19, 29 July 2008 (UTC)[reply]

I agree that the question as stated was inappropriate, but the asker might like to check out a more sensible option at Scleral_lenses#Use_in_special_effects. --Sean 14:53, 29 July 2008 (UTC)[reply]
First you gotta kill a few people. Then you got to get sent to a slam, where they tell you you'll never see daylight again. You dig up a doctor, and you pay him 20 menthol Kools to do a surgical shine job on your eyeballs. Plasticup T/C 16:21, 29 July 2008 (UTC)[reply]

driving licences in ireland

can anyone tell me what year driving licences were given out free in ireland.Bigbuffer (talk) 13:39, 29 July 2008 (UTC)[reply]

Food for thought...

Has there ever been an attempt to fully automate a fast-food restaurant ? They could have customers enter the number of their choice, say a double cheesburger, then select None, Normal, or Extra for each topping, like tomatoes, and a machine could then create it for them. I'd think it might be popular, just due to the novelty. StuRat (talk) 13:55, 29 July 2008 (UTC)[reply]

See automat. They are still very popular in The Netherlands. 195.35.160.133 (talk) 14:11, 29 July 2008 (UTC) Martin.[reply]
Except there's still people in the back making the food. I don't think there's ever been an attempt to fully automate the entire process of preparing all the ingredients and assembling a sandwich at a restaurant level. --Random832 (contribs) 17:21, 29 July 2008 (UTC)[reply]
You might see it if minimum wages are significantly increased, but without that there's really no point. Getting people to do it is far cheaper than designing, building and maintaining robots to do it. --Tango (talk) 01:53, 30 July 2008 (UTC)[reply]
The whole point is that people might go there, versus a human-run restaurant, for the novelty of seeing robots prepare their meal. StuRat (talk) 04:06, 30 July 2008 (UTC)[reply]
We already have something conceptually similar. It's a vending machine for coffee that allows the customer to specify if and how much cream and sugar he/she wants in the coffee. --71.162.242.76 (talk) 17:03, 30 July 2008 (UTC)[reply]
Pizza vending machines. APL (talk) 22:43, 30 July 2008 (UTC)[reply]
That's getting a bit closer to what I meant, although they apparently only have 3 pizzas to choose from, which are then heated. I'd have allowed the customers to specify the toppings and then would have built each pizza to order. StuRat (talk) 00:05, 31 July 2008 (UTC)[reply]

Conservapedia

Does anyone take Conservapedia seriously? Most of the articles just seem so ridiculous that it's hard to work out whether they're genuine or satire inserted by the "liberal" conspiracy. I cannot imagine anyone actually suggesting that it should be used as anything other than a source of mis-information. Jooler (talk) 15:36, 29 July 2008 (UTC)[reply]

There's probably somebody, but I can't imagine who Emma Hordika (talk) 15:52, 29 July 2008 (UTC)[reply]

Poe's Law.[9][10] It's impossible to parody a fundamentalist. --h2g2bob (talk) 16:23, 29 July 2008 (UTC)[reply]

There are an ever-increasing group of Wikipedia knock-offs meant to serve specific agendas (of course, they always posit Wikipedia is the one having the agenda, but anyway). There are Conservative ones and White Supremacist ones and probably others I don't know about. As far as I can tell, they have two audiences: the people who write them (usually a handful of people, no more), and people who are unsavvy about the ways of the internet. For example, one of the slanderous political e-mails going around lately is about Obama's mother, and it looks, superficially, like it came from Wikipedia. (My grandmother even forwarded it to me.) But in reality it's from one of the white supremacist Wikis and is authored by an admitted member of a neo-Nazi group. Was I fooled? No, I'm internet savvy, and I know better than to trust ANYTHING in one of those political e-mails, especially without doing a little bit of research into its origins/veracity first. But my grandmother? She has no clue. She only figured out there was a site called Wikipedia a few months ago and she wouldn't have the slightest idea how it worked. As far as she's concerned any site that calls itself an encyclopedia is set up along the same old lines as her print encyclopedias. She's not dumb, but she's very old, and in her lifetime has watched computers go from something that fills entire rooms to something small and strange on a desk. She, unwittingly, is one of the audiences for these splinter encyclopedias. Which is depressing, I guess. --98.217.8.46 (talk) 16:28, 29 July 2008 (UTC)[reply]

I'm sure many people do; American neoconservatism and the religious right are very powerful cultural movements. Conservapedia is-or was originally-intended as a resource for and by homeschooled children, which may explain the slightly simplistic nature of many of the articles. FiggyBee (talk) 16:31, 29 July 2008 (UTC)[reply]
Neoconservatism and the Christian right should not be conflated; they have some points of commonality but are very different in origins and worldviews. I don't think neoconservatism is a cultural movement at all (in the sense of having much of a footprint in the popular culture--certainly it's a movement that has strong prescriptive views about culture). It's more of an intellectual movement, and basically secular. I doubt that secular neocons have much use for Conservapedia, which really seems to be specifically aimed at the Christian right. --Trovatore (talk) 01:17, 30 July 2008 (UTC)[reply]
I can think of another explaination... Plasticup T/C 18:59, 29 July 2008 (UTC)[reply]
Homeschooling is not synonymous either with fundamentalist Christianity or with simple mindedness. People choose to homeschool for many reasons. Not least of which is that public schools in the US suck right now. Please don’t foster stereotypes here! :) More to the point, I suspect that many of the more extreme articles on Conservapedia have in fact been authored by savvy trolls. --S.dedalus (talk) 21:43, 29 July 2008 (UTC)[reply]
To actually answer your question, yes, Andrew Schlafly does want Conservapedia taken seriously. Just maybe two weeks ago, a former WP bureaucrat and developer failed a RfA, partly because he's a CP admin. As for the liberals, all we got is RationalWiki, which takes itself a little bit less seriously. Paragon12321 (talk) 17:16, 29 July 2008 (UTC)[reply]
I missed that - who was it? (link?) Exxolon (talk) 18:56, 29 July 2008 (UTC)[reply]
See Wikipedia:Requests for adminship/Ed Poor. Graham87 15:02, 30 July 2008 (UTC)[reply]
S.dedalus; I meant to suggest that the simple writing style was suggestive of highschool-age students, rather than homeschooled people. And yes, I wouldn't be surprised. FiggyBee (talk) 23:04, 29 July 2008 (UTC)[reply]

Maybe it'd be taken a little more seriously if he didn't think the Axiom of Choice was a liberal conspiracy. (He doesn't like it, fine, but what the hell's political about it?) --Random832 (contribs) 17:23, 29 July 2008 (UTC)[reply]

lol --S.dedalus (talk) 21:45, 29 July 2008 (UTC)[reply]
Hmm -- I took a look at Conservapedia's AC page and, while there are some fairly weird statements (like the one about "formulating a constructive axiom of choice" being one of "three major problems" for math-logicians), I didn't see anything suggesting that AC is political. --Trovatore (talk) 23:24, 29 July 2008 (UTC)[reply]
After reading Conservapedia's article on the subject, I find it gives a better explanation in layman's terms than the Wikipedia article. − Twas Now ( talkcontribse-mail ) 00:51, 30 July 2008 (UTC)[reply]
See http://www.conservapedia.com/Examples_of_Bias_in_Wikipedia - it gives our insufficiently critical presentation of proofs relying on the AC as an example of liberal bias. He's got a bit of an ideological thing for constructive proofs. http://rationalwiki.com/wiki/Conservapedia:Conservapedian_mathematics (yes, rationalwiki is a bit biased against conservapedia, but the claims seem to check out) --Random832 (contribs) 04:00, 30 July 2008 (UTC)[reply]
Wow. Just in case anyone follows that link and wonders about the use of the axiom of choice in Wiles's proof of Fermat's Last Theorem: It's possible, for all I know, that the proof does make use of AC. However, this should cause no doubt in the minds even of those who think AC is false. If you can prove a result about the natural numbers with AC, then you can also prove it without. (There are some caveats here, such as about the meaning of the word "prove" and whether Wiles's proof meets that standard, but that digression would take us too far into technicalities—suffice it to say that almost certainly no serious anti-AC mathematician doubts Wiles's proof because of the use of AC.) --Trovatore (talk) 04:18, 30 July 2008 (UTC)[reply]

Is Australia really stuck 10 years in the past?

When I last visited Australia it seemed as up to date at the rest of the modern world, so why does the stereotype that Australia is 10 years behind everyone else still persist? Are there any cases where this is indeed still true; fashion, music, industry, etc? Any other countries like this, Egypt, South Africa, Spain? Dark Angel X5 Max (talk) 19:13, 29 July 2008 (UTC)[reply]

No no, they are literally 10 years in the past. It takes a while for space-time to stretch around the curvature of the Earth and reach them down at the bottom. In seriousness, I have never heard of this stereotype. Where are you from? Plasticup T/C 19:53, 29 July 2008 (UTC)[reply]
I've never heard of that stereotype either. --Tango (talk) 20:04, 29 July 2008 (UTC)[reply]
It's certainly an old saw (although when talking about Queensland, the state I live in, we tend to say 50 years!), but maybe not as true as it once was. If you want physical and political historical examples of backwardness; Brisbane had large areas unsewered into the 1970s, and Queensland had laws against homosexuality until less than 20 years ago. There are, I can tell you from experience, very strong currents of racism, homophobia and anti-intellectualism in the idealised (white rural) Australian culture, although in a much more laid-back way than you might see in other countries. FiggyBee (talk) 20:22, 29 July 2008 (UTC)[reply]
Strong, yet laid-back currents of racism, homophobia, and anti-intellectualism? This I have to see! − Twas Now ( talkcontribse-mail ) 00:47, 30 July 2008 (UTC)[reply]
(ec) That anti-intellectualism of which you speak is sometimes not as laid back as you say, FiggyBee, but quite in-your-face, and very disturbing for one of the better educated societies on the planet. It's particularly apparent in TV shows aimed at the yuppie market (anything associated with Rove McManus, for example). His guests and game show competitors make an art form out of professing and celebrating their ignorance, and thinking that ignorance is cause for joy and general merriment. The pity is, they're rewarded (sometimes very handsomely) for this, so it gets reinforced in the minds of viewers that it's not only wonderful to be ignorant, but that it's also not OK to actually know something or give a damn about history or "the finer things of life". The ad about the father telling his son the Great Wall of China was built by Emperor Nasi Goreng in order to keep out the rabbits is another example. That was a bit of fun, and nobody would take it seriously as a history lesson, but many viewers could relate to it, which is why it's been shown for years. -- JackofOz (talk) 01:01, 30 July 2008 (UTC)[reply]
I wouldn't say anti-intellectualism is backward. As far as I can tell, it's quite a modern social problem. In the past people "knew their place" and respected people that were well educated. Now being upper class is considered a bad thing and being well educated is still associated with being upper class. --Tango (talk) 01:34, 30 July 2008 (UTC)[reply]
I don't quite agree about the class thing. There are some people who are very class-conscious, but they're in a decreasing minority. Prime Ministers are very used to being addressed by ordinary people as "Bob", "Paul", "John" or "Kevin" these days (although "Malcolm" was a little aloof and he usually got "Mr Fraser"). Going further back, it would have been unthinkable for a PM to have been addressed, or even referred to in the media, other than as "Mr <surname>", but that's all changed now. Most of the people who publicly profess their ignorance are the very people who do or would send their kids to school and ensure they get a good education so they can do even better than they themselves did. Education is seen to be about economic achievement, not about scaling class walls. -- JackofOz (talk) 01:54, 30 July 2008 (UTC)[reply]
I don't really know about Australia, but in the UK the reduction in class-conciousness has gone so far as to end up going the other way, see reverse snobbery. That's what I was referring to. Anti-intellectualism and reverse snobbery kind of go hand-in-hand due to the (now outdated) association of the upper class with higher education. Maybe it's different in Australia, but in the UK there are a lot of poorly educated parents who do very little to encourage their children to get a good education. They send them to school, but that's about it, and sometimes don't even bother with that much. --Tango (talk) 02:47, 30 July 2008 (UTC)[reply]
I'm also Australian, although I do suppose we would come off as a bit racist, we're overly-PC now, since Rudd and his predecessor still seemed intent on being sheep to the US. Quite sad really.Avnas Ishtaroth drop me a line 03:28, 30 July 2008 (UTC)[reply]
And yes, our humour is rather simplistic, crude and degrading. We did come from convicts after all! And yes, although it is a shame to admit it, we have a minority of homeless and dole bludgers even in the capitals, like Tango said in the UK, some people can't be bothered. My father went to a state school which was mostly made up of deros (derelict/juvenile delinquent) who got into fights a lot. This might have changed a bit now, I don't know the state of things in government schools. Maybe we should reinstitute corporal punishment...my father sure thinks so. Myself, my brother and I agree that we should secede from the rest of this bloody country, they nick all our wealth and resources and give us no credit for it.Avnas Ishtaroth drop me a line 03:43, 30 July 2008 (UTC)[reply]
Should we wait ten years before archiving this, so everyone has an opportunity to respond in the fashion they consider timely? Edison (talk) 23:42, 31 July 2008 (UTC)[reply]
This was asked by the AT. Fewer responses are better and deletion is the best. Zain Ebrahim (talk) 15:09, 1 August 2008 (UTC)[reply]

Can I get back to you on that, say August 1st, 2018? DOR (HK) (talk) 04:30, 1 August 2008 (UTC)[reply]

the mayflower

I couldnt contact you any other ways else. so. Your page about the mayflower is wrong and i cant change it because there is a lock on the page. I have been to where the pilgrims sailed from IN ENGLAND. I am half British. The pigrims sailed from IMMINGHAM england. not southampton. I have touch the rock that is there memorial. LOOK IT UP. thank you. —Preceding unsigned comment added by Kungfu445 (talkcontribs) 20:31, 29 July 2008 (UTC)[reply]

As our articles on both the Pilgrims and Immingham note, Immingham was the departure point for the Pilgrims' migration to Holland in 1608. The departure of the Mayflower to the New World was from Southampton in 1620. FiggyBee (talk) 20:43, 29 July 2008 (UTC)[reply]

Which Desk to use

Where is the best desk to ask a psychology question? Science, humanities, and miscellaneous all seem to be possibles. Thanks, Wanderer57 (talk) 21:39, 29 July 2008 (UTC)[reply]

science or humanities depending on the question.87.102.86.73 (talk) 22:11, 29 July 2008 (UTC)[reply]
Thank you. The question is here:
Wikipedia:Reference desk/Language#Thesarus request
Perhaps you or someone else will suggest a better place to pose the question, or answer it directly.
Thanks, Wanderer57 (talk) 00:27, 30 July 2008 (UTC)[reply]
It depends on how technical of a question you have in mind. If you're looking for the action potential for a type of neuron then the science board is probably the way to go. If you're interested in psychoanalysis then the Humanities board is probably best. I'd put my money on the Humanities board but I check them all and will attempt an answer if I know enough to form a coherent response.--droptone (talk) 12:13, 30 July 2008 (UTC)[reply]


July 30

Are the old reference desk questions saved anywhere?

I had asked a question some time back and got my answer. I wanted to look up the answer again, as I had the same question, but can't find my question with answer anywhere. Is there some archive of asked questions, or is it gone forever? The Mighty KELP (talk) 00:37, 30 July 2008 (UTC)[reply]

There is an archive. What was your question about? Zagalejo^^^ 00:43, 30 July 2008 (UTC)[reply]
From what I can tell from your contribution history, it was either this or this. Zagalejo^^^ 01:04, 30 July 2008 (UTC)[reply]
Yes. You can find old questions in the archives. − Twas Now ( talkcontribse-mail ) 00:44, 30 July 2008 (UTC)[reply]
And by searching here[[11]]. Julia Rossi (talk) 06:03, 30 July 2008 (UTC)[reply]

Heads-up

Not a question: Please see Government Service Insurance System.

First paragraph not factual. —Preceding unsigned comment added by 160.83.160.132 (talk) 00:49, 30 July 2008 (UTC)[reply]

Thank you for your suggestion. When you feel an article needs improvement, please feel free to make those changes. Wikipedia is a wiki, so anyone can edit almost any article by simply following the edit this page link at the top. The Wikipedia community encourages you to be bold in updating pages. Don't worry too much about making honest mistakes — they're likely to be found and corrected quickly. If you're not sure how editing works, check out how to edit a page, or use the sandbox to try out your editing skills. New contributors are always welcome. You don't even need to log in (although there are many reasons why you might want to).
Either that, or tell us what's wrong about it. --Tango (talk) 01:26, 30 July 2008 (UTC)[reply]
Possibly solved with this edit. − Twas Now ( talkcontribse-mail ) 01:32, 30 July 2008 (UTC)[reply]
Ah, yes, looks like you got it! --Tango (talk) 01:50, 30 July 2008 (UTC)[reply]

Homophile

Someone called me a homophile the other day. what is homophile? —Preceding unsigned comment added by 79.76.156.194 (talk) 01:24, 30 July 2008 (UTC)[reply]

See -phil-. I guess they mean you like homosexuals (not necessarily in the sense of being a homosexual) - ie. the opposite of a homophobe. --Tango (talk) 01:28, 30 July 2008 (UTC)[reply]
Homophile is also an old-fashioned synonym, or a kind of euphemism, for homosexual. In its original meaning, it refers to a person who loves others of his or her own sex, without implying actual sexual relations. Marco polo (talk) 01:42, 30 July 2008 (UTC)[reply]
According to the Oxford American Dictionary, the world "homophile" can also refer to a gay rights activist. --CalusReyma (talk) 10:54, 31 July 2008 (UTC)[reply]

Friendly, open and unbiased. DOR (HK) (talk) 04:31, 1 August 2008 (UTC)[reply]

software/word finder

could you recommend a free software or a word game in which, if I enter the letters of a word it must show the possible words. e. g.1 if I enter the letters a, c, e, e, h, r and t (in alphabetical order ) it must show the word teacher. e. g.2 if I enter the letters d, e, o,r, and r it must show the word order.thank you124.43.49.154 (talk) 05:06, 30 July 2008 (UTC)[reply]

Excellent for cheating at puzzles Someoneinmyheadbutit'snotme (talk) 05:45, 30 July 2008 (UTC)[reply]
Or this one, except that it comes up with more than one possibility for a, c, e, e, h, r, t. --Richardrj talk email 05:49, 30 July 2008 (UTC)[reply]
The word builder on Scrabble.com does the same thing, which was very useful for cheating at Scrabulous. Adam Bishop (talk) 13:42, 30 July 2008 (UTC)[reply]
Or try Anagram Artist. Used it some years back and it was great. Screenshot looks like it is still the same. download here.--196.207.47.60 (talk) 15:58, 30 July 2008 (UTC)[reply]

Tax on Electronics

How much is the Tax for Electronics in Indiana? I'm planing to buy a Notebook there, so how much tax do i need to pay on it??is there any tax rebate for college students??is there a complete tax rebate as i have heard, on Labor day?thnx...cheerio! —Preceding unsigned comment added by 123.252.229.3 (talk) 09:07, 30 July 2008 (UTC)[reply]

Indiana sales tax is 7% now, with possible increases in particular counties or cities. There's almost certainly no exemption for electronics, college students, or Labor Day. Such exemptions are generally reserved for necessities like food. — Lomn 13:24, 30 July 2008 (UTC)[reply]
Not true, Lomn, various exemptions are available. For an exemption for eletronics in Indiana, the equipment must be sold by the college to a student and be required by a curriculum[12]. Several states have a back-to-school sales tax holiday around Labor Day, some of which include electronics. Indiana does not appear on this list of this year's, however: [13] Rmhermen (talk) 16:09, 30 July 2008 (UTC)[reply]
For the record, many states do now have some type of Sales Tax holiday, often around this time of year, and several of them exempt computer purchases from sales tax during the designated period. In addition, in states where computer purchases are not specifically exempted by law, many retailers offer additional discounts to be the equivalent of "no sales tax." However, Indiana does not appear on the linked list, and I was unable to turn up any evidence of such a holiday for the Hoosiers. --LarryMac | Talk 16:07, 30 July 2008 (UTC)[reply]

Hair cuts

Hi there. I'm a guy and I've been trying to grow my hair long for a while now. But every time it gets sorta long it goes frizzy and horrible and I have to cut it. Mine is dark brown and curly. So, what is the best way for me to get nice long straight hair? I'm kinda embarrassed to go to the hair dressers and ask and I don't want to come out with some sorta girls haircut. Turbotanker (talk) 09:17, 30 July 2008 (UTC)[reply]

How long is "sort of long"? I grew my hair for a few years and if you don't want it to get frizzy, tangled and horrible it can take a lot of maintanance; frequent and thorough washing and a lot of brushing and combing. In the end I gave up and went back to my buzzcut. :) FiggyBee (talk) 09:26, 30 July 2008 (UTC)[reply]
Yeah, but I figure I might as well try it long before it all falls out, right? I guess quite long is what I'm looking for, maybe shoulder length to middle of back. When I grew mine out I washed it to begin with, but washing it actually made it more frizzy. I brushed it often but still it just ended up in one hell of a flyaway mess. It wasn't clumping together but it was just all over the place. What I don't want is something like this; that's sorta how mine was going. I want it long and straight like this or this. Any suggestions? Thanks everyone for your help. Turbotanker (talk) 09:42, 30 July 2008 (UTC)[reply]
If it's as curly as the first pic, I don't think that you'll be able to get it as straight as in pics two and three. But you can try. My hair, until recently, was as long as that seen in the first pic though nowhere near as curly. I suggest you look up a salon in the phone book, then call them and ask if they work with male customers. They'll be able to give you a better idea as to whether or not you can get your hair as straight as you like. Straightening hair can damage it, so they'd really be the best people to ask. i.e. someone who has seen it and has experience with these things. Dismas|(talk) 09:59, 30 July 2008 (UTC)[reply]
So, let me get this straight: rather than ask an experienced professional of the hair care industry, who's paid to give you good advice, you'd prefer the advice of some random people on the internet? =) Seriously, though, this is a little akin to medical advice in that none of us can actually see your hair. I suggest you tell a hair dresser that you'd like your hair to be long and ask how to go about it -- and it's perfectly okay and ordinary to bring a picture for demonstration purposes, so there's no risk of misunderstanding. I mean, this is what these people do for a living, and you're going to have to discuss it with someone who cuts your hair anyway if you want it to look neat, so you might as well bite the bullet. (You can't just keep growing your hair out and expect it to look nice and neat without the occasional haircut; otherwise it'll look uneven and start fraying and whatnot. That's not a problem if you're going for some kind of a rock'n'roll look, but the example pictures show that that's not the case.) -- Captain Disdain (talk) 10:05, 30 July 2008 (UTC)[reply]
Thanks everyone, that response is what I expected anyway; "no one here can help you so go to the hair dressers", so I guess I'll have to bite the bullet. BTW Captain Disdain, no pun intended with "So, let me get this straight", eh? Turbotanker (talk) 10:13, 30 July 2008 (UTC)[reply]
You fill me with shame. =) -- Captain Disdain (talk) 13:41, 30 July 2008 (UTC)[reply]
If we can believe the ads there's nothing hair products (no frizz, frizz ease, smoother, shine enhancer, etc) can't fix along with tips from the hair people themselves. But definitely not frequent washing. (medical advice? geez, it's hair as in cosmetic, not cosmedic I hope) Good luck, Julia Rossi (talk) 12:15, 30 July 2008 (UTC)[reply]
1 loads of conditioner 2 right type of conditioner, to eliminate frizz 3 apprently one can get hair relaxer, as used by women of african descent to quote unquote straighten thier hair. anyway, I have hair like in your first pic, and I am quite happy with it. so 4 keep the faith and keep it long. —Preceding unsigned comment added by 193.115.175.247 (talk) 13:01, 30 July 2008 (UTC)[reply]
When hair gets long enough, the weight can help to keep it down. Gathering it up in the back as a pony-tail can help, until then. StuRat (talk) 14:56, 30 July 2008 (UTC)[reply]

Hot hair straighning tongs work for me hotclaws 21:35, 30 July 2008 (UTC)[reply]

Kyuto

Osaka and Kyoto t-shits. I've seen alot of them, and I don't personally like them, but that's just me. I saw a "Kyuto" one the other day, and I was so sure it wasn't a place. Wikipedia comes up with the "This page does not exist" when you put it in, not even a redirect to Kyoto. But Google comes up with weather forcasts for what appears to be a tiny island.

Is it a place too small for notabillity, or is it a colloquial name for another place? Cheers folks. Fenton Bailey (talk) 11:37, 30 July 2008 (UTC)[reply]

The island appears to be Minami Daito. Don't know where the "Kyuto" name comes from. FiggyBee (talk) 11:56, 30 July 2008 (UTC)[reply]
Could be kyūto = cute, I suppose. -- BenRG (talk) 12:11, 30 July 2008 (UTC)[reply]
Thinking the same thing. (snap) Julia Rossi (talk) 12:17, 30 July 2008 (UTC)[reply]
Kyūtō means hot water in Japanese (see EcoCute article). Admiral Norton (talk) 19:50, 30 July 2008 (UTC)[reply]
Is it within the realms of possibility that it belongs to a Japanese coprophiliac?  :) -- JackofOz (talk) 22:59, 30 July 2008 (UTC)[reply]

BlackBerry Phones

Is it possible to get a BlackBerry Phone without the BlackBerry service? like ATT provides an offer without a Data Plan, but then there is a BlackBerry service too if I'm not wrong!The service that enables us to use the special BlackBerry button on the Phone!Don't we need to pay for that service too to make it possible to use the phone? Or is it not required??? —Preceding unsigned comment added by 123.252.229.3 (talk) 14:12, 30 July 2008 (UTC)[reply]

driving records

How do I find out what the longest amount of years someone has driven without a moving violation? I have a friend that has driven(so far) for 60 years with no moving violation. —Preceding unsigned comment added by Bashie (talkcontribs) 15:09, 30 July 2008 (UTC) bashie —Preceding unsigned comment added by Bashie (talkcontribs) 15:13, 30 July 2008 (UTC)[reply]

So has my father, now 89 years old. There is nothing remarkable about not having a traffic violation is there? This would apply to millions of people, such as oldest people.--Shantavira|feed me 15:21, 30 July 2008 (UTC)[reply]
Also wouldn't it be fairer to compare miles driven V driving-offences committed (well been caught doing) rather than length of time? My mother has had a license for 20+ years without an offence, but in that time she will have driven hardly any miles in comparison to a truck-drive who has only been driving, say, 5 years. Obviously it'd be much harder to compare miles-driven rather than time, but I don't think years-driving is a particularly good barometer for this. 194.221.133.226 (talk) 15:35, 30 July 2008 (UTC)[reply]
As well, one would want to look at the jurisdiction involved. The level restrictiveness of traffic laws varies extraordinarily widely from country to country, and the degree to which existing laws are enforced varies from city to city and even from neighbourhood to neighbourhood. I am quite certain that I commit technical violations of the law nearly every time I get behind the wheel, but those violations are seldom observed by law enforcement personnel, and virtually never represent actions which increase the level of danger to myself or to others on the road. (My lawbreaking is mostly confined to borderline speeding, and making the odd rolling stop at an intersection that has good visibility in all directions.) It's always worth remembering that "Didn't get ticketed" doesn't equate to "Drove legally", and has only the most casual acquaintance with "Drove safely". TenOfAllTrades(talk) 18:39, 30 July 2008 (UTC)[reply]
Well, that certainly does apply to my home country. You can legally speed up to 25 km/h (15 mph) on highways and expressways in Croatia without getting fined and you can usually speed up to about 40 km/h and not stick out. Admiral Norton (talk) 19:43, 30 July 2008 (UTC)[reply]
It's not unheard of for drivers at my work to log a million kilometers without an accident. Not an impossible accomplishment for career mid-range truck drivers, but probably beyond the reach of all but the most insane commuters. Matt Deres (talk) 19:50, 30 July 2008 (UTC)[reply]

"Qrre" Stands For (Qualifying Revolving Retail Exposure)

Hello,

I need to understand the (detail) meaning of above acronym, please any one or business related person make me clear the word "qrre" by giving an example, i will be really thankful to you.


Sources:

http://www.fsa.gov.uk/pubs/international/qrre.pdf

http://acronyms.thefreedictionary.com/QRRE

http://www.acronymfinder.com/Qualifying-Revolving-Retail-Exposure-(QRRE).html

(Joyzy (talk) 18:19, 30 July 2008 (UTC))[reply]

This involves the risk ratings that banks assign to their individual assets to determine the capital they must set aside to cover losses. QRRE's are things like credit-cards and overdraft protection given to individuals. Qualifying means the product is a QRRE, Revolving means it's not a fixed loan, Retail means it's to people not companies. Exposure is the amount at risk. The rationale seems to be that QRRE's have a relatively stable "loss rate", i.e. 6% of all credit-card holders default or such-like - so putting them all into one class simplifies the calculation of the amount of money the bank can expect to lose. That's what I gather from the first link you provide anyway. Franamax (talk) 18:41, 30 July 2008 (UTC)[reply]

Thanks Franamax, much clear, anyone else please? —Preceding unsigned comment added by Joyzy (talkcontribs) 06:17, 31 July 2008 (UTC)[reply]

contacting author of an article

Greetings:

I wish to contact the author of the following online Wikipedia article.

http://en.wikipedia.org/wiki/Three_strikes

While all the material is essentially correct, there is one small error I wish to supply to the author so he or she could make the correction.

Thanks in advance,

Dave Three Strikes Webmaster —Preceding unsigned comment added by Decombs21 (talkcontribs) 21:26, 30 July 2008 (UTC)[reply]

Please post such concerns at Talk:Three strikes law (same note left at user's talk page) — Lomn 21:41, 30 July 2008 (UTC)[reply]
Or be bold and just make the change yourself. Plasticup T/C 22:38, 30 July 2008 (UTC)[reply]
Thank you for your suggestion. When you feel an article needs improvement, please feel free to make those changes. Wikipedia is a wiki, so anyone can edit almost any article by simply following the edit this page link at the top. The Wikipedia community encourages you to be bold in updating pages. Don't worry too much about making honest mistakes — they're likely to be found and corrected quickly. If you're not sure how editing works, check out how to edit a page, or use the sandbox to try out your editing skills. New contributors are always welcome. You don't even need to log in (although there are many reasons why you might want to). --Tango (talk) 22:41, 30 July 2008 (UTC)[reply]
Hi. Also, there usually is no single author of an article. Anyone can improve and write articles, including yourself. In fact, there are sometimes hundreds or even thousands of people who have helped write an article, so don't be afraid to improve it yourself. Thanks. ~AH1(TCU) 20:51, 31 July 2008 (UTC)[reply]

Growing new teeth

Don't worry, I'm not pregnant ;-). I am having my wisdom teeth out in two weeks. They are impacted and pushing my other teeth together. This is uncomfortable. Obviously I have seen a dentist and the solution is to have the (wisdom) teeth removed, but in the meantime do what do wikipedians do for their achy, squeezed teeth? Plasticup T/C 21:49, 30 July 2008 (UTC)[reply]

Not a whole lot. There are topical anesthetics you can buy at the drug store (at least in the US) which will temporarily numb your teeth and gums but that's about it. --98.217.8.46 (talk) 21:57, 30 July 2008 (UTC)[reply]
You doctor may prescribe pain killers if you ask. There are also plenty of over-the-counter pain killers available - your pharmacist can advise you there. I can't say any more than that though - you should know better that to ask for medical advice, even if you aren't pregnant! --Tango (talk) 22:45, 30 July 2008 (UTC)[reply]
You should ask your doctor, dentist, or pharmacist for suggestions. They're all trained to offer guidance in the correct use of prescription and over-the-counter medications, and can counsel you on their appropriate use. (Seriously—pharmacists spend a lot of time in school and get paid a lot of money to do what they do. To not take advantage of that training would be silly. And you have to go to the pharmacy to get your painkillers anyway, so it's not going to be out of your way.)
Any specific suggestions we could offer here would be reckless and irresponsible, since few of us have any clinical training, and none have had the opportunity to either examine you or to discuss your medical history. TenOfAllTrades(talk) 22:55, 30 July 2008 (UTC)[reply]
If we were to recommend a prescription, perhaps. However, non-prescription drugs (and non-drug treatments) don't require the recommendation of a doctor, pharmacist, or other professional, so this isn't stepping on anyone's toes. StuRat (talk) 23:54, 30 July 2008 (UTC)[reply]
I second that comment about pharmacists - they are extremely knowledgeable people and are seriously under used by the general public. They're generally trained in some non-pharmaceutical matters as well - they can help with all kinds of medical questions (they'll know which ones they aren't qualified to answer and will tell you to see a doctor if you need to). --Tango (talk) 01:42, 31 July 2008 (UTC)[reply]
The medical advice bit has already been covered. The dentist has been consulted and his solution (extraction) will be performed in 14 days. I wasn't looking for medical advice, just a bit of information on toothaches. Do you drink ginger tea, suck an ice cube, that sort of thing? Mothers remedies, not real medical advice. And I would like to point out that the RefDesk guidelines are that you shouldn't give medical advice, not that I can't ask. If you feel that you cannot answer my question without coming into conflict with that guideline then it is probably best to avoid the question entirely. Others will be able to reply appropriately in your stead. Plasticup T/C 23:49, 30 July 2008 (UTC)[reply]
Menthol cough drops can help. Don't overdo it, though, or your whole jaw will go numb. StuRat (talk) 00:00, 31 July 2008 (UTC)[reply]
I don't know of any folk remedies that will help. If you call your dentist about the temporary problem, they can recommend a product proven effective in reducing dental pain, either OTC or they will write you a prescription for the fast-acting version that makes it all fine right quick. A pharmacist will also tell you the right painkiller and the best topical anaesthetic. I don't want to harp on the subject, but the information on toothaches I would give you is don't bother with folk remedies, go to the pros and do it right - it's not worth the bother of waiting to see if the latest idea will work when your jaw is clanging, I learned that the hard way. Five dollars will solve your problem and the pro will ask you other questions that might reveal a big honking abscess or some other new development. As to your pregnancy, you can't deny it forever you know ;) Franamax (talk) 01:30, 31 July 2008 (UTC)[reply]
The medical advice for the long term problem has been covered, you're asking up for medical advice for the short term problem of pain - that's still a medical issue. You can try all kinds of folk remedies, but I doubt any of them are anywhere near as effective as pain killers. --Tango (talk) 01:42, 31 July 2008 (UTC)[reply]
I don't want to spark an argument right here, but someone ought to start a discussion at the village pump (or wherever it would be appropriate) about writing more specific guidelines on what is and isn't medical advice. This question and the one a few days ago on the science refdesk (and the responces they provoked) seem to mandate it.--El aprendelenguas (talk) 01:03, 1 August 2008 (UTC)[reply]

Suggest you Google and find out about wisdom tooth extraction. It is not an easy extraction (unless unconscious) and I found it helpful to understand so I could co-operate.86.202.155.214 (talk) 14:08, 31 July 2008 (UTC)DT[reply]

Suggest that if the professional who is planning an extraction in 14 days can't deal with your pain in the meantime then find another. -hydnjo talk 00:59, 1 August 2008 (UTC) addendum: Oh, and a toothache means that its a boy! -hydnjo talk 01:02, 1 August 2008 (UTC)[reply]

Rubber bands on braces

What is the purpose of those little rubber bands on braces? --75.62.199.77 (talk) 22:37, 30 July 2008 (UTC)[reply]

I mean dental braces. --75.62.199.77 (talk) 22:37, 30 July 2008 (UTC)[reply]

Presumably they are to place tension on the teeth while still allowing some movement. --Tango (talk) 22:46, 30 July 2008 (UTC)[reply]
And the tension is supposed to pull the teeth back into the proper alignment. It's amazing that some tiny rubber bands can do that, but, given time, they can. StuRat (talk) 23:46, 30 July 2008 (UTC)[reply]
Translation for Brits: "rubber bands" = "elastics". StuRat (talk) 23:46, 30 July 2008 (UTC)[reply]
Um, which way? I'm British and I'd call them "rubber bands". "Elastics" doesn't sound right. 81.187.153.189 (talk) 00:14, 31 July 2008 (UTC)[reply]
I'm British and would say either "rubber bands" or "elastic bands" (the two being interchangeable), never "elastics". --Tango (talk) 02:59, 31 July 2008 (UTC)[reply]
Perhaps it's an Aussie/Kiwi or other dialect where the word "elastics" is used. Oh well, as long as you don't call them "rubbers". :-) StuRat (talk) 13:24, 1 August 2008 (UTC)[reply]
Nope, they're rubber band down here, too. KiwiBiggles (talk) 05:32, 2 August 2008 (UTC)[reply]
There are two different types of rubber bands on dental braces (says someone who had a decades' worth of orthodontia). There are the ones that are on the front of the teeth. They hold the braces to the wire. They are very small. The teeth themselves have little hooks cemented to them, then the wire is held on with the rubber bands. They are very tiny but very strong. The advantage to using them is that you can remove them and then reposition the wire over time—this happens one a month or so, as the orthodontist sees need for putting more pressure in certain directions on certain teeth. These are often colored (though if kids are smart, they eventually realize that silver colored ones look the best and call the least unwanted attention to the braces themselves).
The other types of rubber bands are larger and are attached to bigger hooks usually in the back of the mouth. They, for example, can pull your lower jaw in a given direction. They are more intense and if I recall you usually don't keep them in when you eat (I think for awhile I only wore mine at night?). --98.217.8.46 (talk) 01:21, 31 July 2008 (UTC)[reply]
I currently have braces, and I have a thing like this [14] that goes all the way around over my lower wire. The purpose is to create more tension and close gaps faster than the wire could on its own. As for colouring, yes I have noticed that the rubber stains slightly, but since I have transparent brackets and white wires, I'm sticking to the standard semi-transparent rubber. FiggyBee (talk) 03:24, 31 July 2008 (UTC)[reply]
In my experience the transparent ones end up refracting light funny and staining. Of course, this was 10 years ago. But I'm tellin' ya, going for "silver" colored will look snazziest! ;-) --98.217.8.46 (talk) 13:13, 3 August 2008 (UTC)[reply]
Possibly if I had silver-coloured brackets and wires, but like I said, I don't. :) FiggyBee (talk) 13:24, 3 August 2008 (UTC)[reply]

2012

According to The Bible Code, an asteroid would collide with the Earth in 2012. How likely is this? How accurate is The Bible Code? How accurate are other prophecies that predicted doomsday in 2012? 67.150.127.89 (talk) 23:12, 30 July 2008 (UTC)[reply]

For the likelihood of this happening, I would start with reading Impact event and go from there. As to the accuracy of the Bible Code, well, I think that is more a question of faith than anything else. - EronTalk 23:22, 30 July 2008 (UTC)[reply]
The chance of a large asteroid colliding with Earth in 2012 is very very low - we would almost certainly have spotted it by now. There's a chance a small asteroid could collide, I suppose, but not one large enough to be considered "doomsday". We have an article: Bible code. I expect it will give you some information about the reliability of the code. --Tango (talk) 23:52, 30 July 2008 (UTC)[reply]
We already had a 2012 discussion. Avnas Ishtaroth drop me a line 01:12, 31 July 2008 (UTC)[reply]
The Bible Code can find anything you want to find in it. It's not "accurate" in the sense that it has predicted anything that has actually happened (though you can find past "predictions" in it after the fact, because you can find anything you want in it). You can use pretty much any book in any language that is long enough, and you'll find all sorts of apparently meaningful sequences in it. It's bunk. --98.217.8.46 (talk) 01:17, 31 July 2008 (UTC)[reply]
I hate to malign the Bible Code and "other prophecies," but a lot of prior predictions have apparently been less than accurate, as evidenced at The Millenium and TEOTWAWKI. Just this year, we've escaped disaster predicted for March 21 (Armageddon failed to occur), April 6 (start of the millennium of Jesus), April 17 (the seventh seal [yet again]), May (no specific date, but bar codes didn't destroy us)... you get the idea. OtherDave (talk) 01:19, 31 July 2008 (UTC)[reply]
We already had two 2012 discussions. -LambaJan (talk) 02:10, 31 July 2008 (UTC)[reply]
Asking how accurate a prediction is is always impossible to answer, because nobody knows the future. The only way to know whether or not it's accurate is to wait until the predicted time and see what happens, or not. If I predicted "the sun will rise (as in, appear over the eastern horizon) tomorrow morning", you might say that's a very accurate prediction because everyone knows it will happen, with virtually 100% certainty. But one of these days it may not rise, and that day just might be tomorrow. -- JackofOz (talk) 04:07, 31 July 2008 (UTC)[reply]
Of course. Did anyone suggest judging the accuracy of a prediction before the given time had passed? --Tango (talk) 05:07, 31 July 2008 (UTC)[reply]
Well, since you ask, as a matter of fact they did. How accurate is The Bible Code? was one of the questions. Was the questioner asking about the general record of accuracy of the various predictions contained in the book (I've never read so I'm going on what I've heard of it), or was the question specifically related to the prediction about the asteroid? I think the latter, but I suppose the former would not be an unreasonable interpretation. If it was the latter, was the questioner expecting to wait till 2012 before they got an answer from us? Not bloody likely. They want an answer now, but we simply don't know now. In the meantime one could, by making various assumptions and doing a fair bit of scientific research, come up with an estimate of the likelihood that the event will happen as predicted, but accuracy can only be determined by whether it actually happens or actually doesn't happen in 2012, and there's no way to know that until, well, 2012. This question is in the same vein as "Will Heath Ledger be posthumously nominated for an Oscar?" or "Will Obama win in 2008?". We can talk till kingdom come about likelihoods etc, but the only way to know for sure is to wait and see. -- JackofOz (talk) 05:42, 31 July 2008 (UTC)[reply]
I think one can say that if something has failed to make any predictions of things that have actually occurred in the past, then it is unlikely to start doing so in the future. It's obviously not bullet-proof logic but as a heuristic for predictions it works pretty well at filtering out the total B.S. --98.217.8.46 (talk) 13:42, 1 August 2008 (UTC)[reply]
And for pure entertainment there are topics related to the Doom series. Julia Rossi (talk) 09:57, 31 July 2008 (UTC)[reply]

Um, don't asteroids collide with the Earth every year? --Kjoonlee 03:19, 2 August 2008 (UTC)[reply]

Oops, no. Asteroids are bigger than meteoroids by definition. --Kjoonlee 07:47, 2 August 2008 (UTC)[reply]
Asteroid 2004 VD17 has an impact probability of 1 in 41.667 million in the next 100 years. Seems to be the only really big asteroid with any chance of hitting earth for a while. --S.dedalus (talk) 01:02, 3 August 2008 (UTC)[reply]

LEGO CUSTOMISING

anyone know how can i change the color of my lego minifigures ( without changing their shapes or distorting therir shape ) what kind of paint shall i use ... —Preceding unsigned comment added by 88.227.51.130 (talk) 23:31, 30 July 2008 (UTC)[reply]

Model paint is usually oil based, like enamel or whatever kind of paint is used for Warhammer figures which are plastic too. Some current acrylics may be okay as well, just keep it thin as possible. Julia Rossi (talk) 09:51, 31 July 2008 (UTC)[reply]
I'd use enamel type because I don't think acrylic paints stick well to smooth plastic surfaces, and would probably just come off if scratched. Unless you want to sand the surfaces a bit to make the surface rougher, which is probably not a bad idea for either choice anyways, if you're not planning on re-using the pieces. --Wirbelwindヴィルヴェルヴィント (talk) 18:08, 31 July 2008 (UTC)[reply]
You may consider dipping the figurines BRIEFLY into a shallow bowl with paint stripper. This should give the plastic surface a bit of "bite" and make the paint adhere much better. Unless you use something like a dentist´s drill, sanding the little shapes is rather awkward.
Make a test on some throw away item first to determine the proper time / dilution or you may be left with an amorphous blob of sticky plastic. Also wear goggles and gloves, don´t smoke and stop breathing whilst working.
Like WW above, I would use enamel, as it dries from the bottom up. Acrylic paint dries from the surface down and whilst you may think it is dry, the film underneath is still tacky (and probably will be for a long time, as it can´t dry out properly). The whole paint may crinkle up when you touch the pieces.
There are also some special paints available in shops for model trains but I don´t know how they work when used on plastic surfaces. You may also need a reducer (for the paint) and a plastic adhesion promoter as a primer.
Clean the figures with a toothbrush or the like. Deposits of sweat and grease from your fingers will muck up your paint job. --Cookatoo.ergo.ZooM (talk) 19:45, 31 July 2008 (UTC)[reply]

July 31

betting on sportbet.com

Does anyone here bet on sportbet.com? I tried to register for it, but don't know where I can make bets for college football picks? Does anyone know where I can place over/under bets for spreads on that site?76.212.141.163 (talk) 04:36, 31 July 2008 (UTC)[reply]

That's not the kind of question the Ref Desk is intended for - if you're having problems with a specific site, there should be a way for you to get help on that site. Look for a "contact us" link or something. --Tango (talk) 05:06, 31 July 2008 (UTC)[reply]

Yes, but I searched the website, and didn't find an answer to my question. That's why I'm asking it here? And doesen't the Miscellaneous section answer any questions anyways?76.212.141.163 (talk) 07:09, 31 July 2008 (UTC)[reply]

Sometimes we also use google[15]. Julia Rossi (talk) 09:53, 31 July 2008 (UTC)[reply]
The ref desk answers the kind of questions you would find the answer to in a library, it's not a generic help forum. If you can't find the answer on their website, we probably won't be able to either, so it would rely on one of us already knowing the answer. There are far better places than this to find people that already know about sportbet.com (google will help you find some, I'm sure). --Tango (talk) 16:48, 31 July 2008 (UTC)[reply]
FWIW, if I find a website is so badly designed that I can't find answers on how to use it, I wouldn't trust them with my money. There are plenty of other betting websites who will only too happily take your cash.--Shantavira|feed me 17:27, 31 July 2008 (UTC)[reply]

Peel P50 fuel tank capacity

Hi everyone, I'm looking to buy a Peel P50, and I would like to know the specifications before I do so - however, no matter how hard I look I cannot find the fuel tank capacity, to figure out the range of the car!! If there is anyone who knows, or knows where to find it, please help!

144.138.21.113 (talk) 07:12, 31 July 2008 (UTC)Will[reply]

Hi 144.138, the article Peel P50 says it has a 49 cc DKW engine if you can extrapolate from that. There are some links and then at DKW some stats for a two-stroke. Julia Rossi (talk) 09:43, 31 July 2008 (UTC)[reply]

One of the Googled to websites says it has a tank of 2 gallons, giving you about 200 miles, which at the top speed should then last for 6 hours. If you are a subject of the Lord of Man, who, of course, is neither a man nor possesses 3 legs, this should permit a few shopping trips to the white spots on the map of the island. --Cookatoo.ergo.ZooM (talk) 16:38, 31 July 2008 (UTC)[reply]
And at the going price I'd look for a really secure way to chain it to bicycle racks to deter bag-snatchers from being 200 ks away before you notice. Julia Rossi (talk) 02:09, 1 August 2008 (UTC)[reply]

Solar eclipse

I live in Tashkent, so will it be possible to me to observe tomorrow's solar eclipse? 89.236.214.174 (talk) 08:38, 31 July 2008 (UTC)[reply]

From this NASA map it appears that the path of totality will be some distance to the East of you, on the far side of Kazakhstan. So from Tashkent you will see a partial eclipse but not a total eclipse. Gandalf61 (talk) 09:28, 31 July 2008 (UTC)[reply]
That same site has a Google map you can manipulate, with lines to show the path of the eclipse. OtherDave (talk) 09:33, 31 July 2008 (UTC)[reply]
Thank you :) 89.236.214.174 (talk) 09:44, 31 July 2008 (UTC)[reply]

job search in London

I want to move to London and search for a job there. At the beginning I would like to live near London, but in a place that is cheaper. Any suggestions for a place that is not a rip-off and is well connected to London? Any other random suggestion relating to job search in London?

You might like to browse Commuter town for the London area under the Effects section until someone who knows comes along. Julia Rossi (talk) 09:45, 31 July 2008 (UTC)[reply]
Thanks for the prompt answer. I am impatiently waiting for more. —Preceding unsigned comment added by 81.47.146.188 (talk) 09:54, 31 July 2008 (UTC)[reply]
Take a look at commutefrom.com. It provides an interactive map that lets you filter main-line stations by commuting journey time to various locations in cental London, and provides links from each location to the BBC UK house prices database and other sources of local information. Bear in mind that you will need to budget for higher travel costs if you are commuting. Also travelling to the station from which you are commuting will add to your journey time and if you will be driving to the station, don't forget car parking costs. Gandalf61 (talk) 10:28, 31 July 2008 (UTC)[reply]
Don't forget also that some parts of London are cheaper than some places that are further out. Croydon is not a bad compromise, it's not a particularly trendy place but it has excellent rail links to the capital, plenty of shops and a nice light rail system for getting around. --Richardrj talk email 10:48, 31 July 2008 (UTC)[reply]

watford

This website has a list of average rental prices per week for London postcodes and will at least give you an idea of areas and how much you can expect to pay. It only covers London postcodes though, plus a few others. So outer London ones Croydon and Watford as suggested here (both of which I have lived in/near and would recommend for having lowish costs and decent commutes to the centre) are not on this list. Franmars (talk) 13:56, 31 July 2008 (UTC)[reply]

We lived in Croydon for some 10 years. There are major benefits: Rail: Victoria 17 minutes, Gatwick airport 35 minutes, soutyh coast about an hour. Very good shopping, reasonable nightlife. Good College. Big ethnic mix. Light tramway gets one about. Parking is dreadful and Council will clamp anything that does not move for 30 seconds. But cheap and London do not go together. However a London job should pay London rents. But London jobs are hard to find, thousands are looking. Good luck.86.202.155.214 (talk) 14:06, 31 July 2008 (UTC)petitmichel[reply]

This site displays average rental prices for all UK towns and counties, and it considers unitary authorities (e.g. Luton) and Greater London boroughs as counties. Only the county data is broken down by size of rental unit, but you can compare the overall average for a town, such as Watford, with the overall average for its county (in this case Hertfordshire), to generate a proportion (town average rent as a percentage of county average rent). You can then apply this proportion to the average rent for the size unit that interests you in that county to get a rough estimate of the average rent for the same size unit in the town that interests you. Marco polo (talk) 19:10, 31 July 2008 (UTC)[reply]
By the way, the London borough of Barking and Dagenham seems to have the lowest average rents in Greater London, and Barking seems to have fairly good rail and underground connections to central London, but I do not live in the area and know nothing about the quality of life there. Marco polo (talk) 19:23, 31 July 2008 (UTC)[reply]
The cheaper housing tends to be in the east, not just in London but over the border including places like Grays and Basildon. However, rents rise the closer the flat is to a railway station, because most London workers commute by rail. London's employment centres will be different depending on the type of job you want - the newer financial services and a lot of corporate offices are in Docklands which is east of centre; legal jobs are mostly around Holborn; retail jobs are centred on Oxford Street, and civil service jobs are mostly in the south. If you're looking for a job which does not require specialist skills, then make sure you are among the first to get to the JobCentre Plus in the morning. Sam Blacketer (talk) 13:28, 1 August 2008 (UTC)[reply]

Northernmost point in Russia?

Does anyone know what is the northernmost point in Russia? (reachable by car, or on foot) Thank you! --RiseRover|talk 13:20, 31 July 2008 (UTC)[reply]

They've claimed the north pole since 1926. Other than that, see Extreme points of Russia. --Random832 (contribs) 13:36, 31 July 2008 (UTC)[reply]
So one answer to your question is Cape Chelyuskin, which you could reach on foot at any time of year, though it is more than 2,000 kilometers from nearest road connected to the Eurasian road network, and you'd have to ford or swim across some major rivers to reach that point. During the winter, with the proper gear, you could get there more easily on foot. (You could walk, with some risk, across frozen rivers.) You might even be able to walk across the sea ice to Cape Fligely. Marco polo (talk) 13:54, 31 July 2008 (UTC)[reply]

skunks spray

How old are skunks when they can spray? I know they are born with the glands but are they able to spray? 142.177.121.251 (talk) 22:20, 31 July 2008 (UTC) Charlotte[reply]

Seven weeks, according to this. Zain Ebrahim (talk) 22:59, 31 July 2008 (UTC)[reply]

August 1

Tiny summer isles

the tiny summer isles are a few miles northwest of which scottish port149.254.120.136 (talk) 10:01, 1 August 2008 (UTC)[reply]

See Summer Isles and Loch Broom. Gandalf61 (talk) 10:11, 1 August 2008 (UTC)[reply]
That would be Ullapool laddie.--Shantavira|feed me 11:52, 1 August 2008 (UTC)[reply]
unless you mean the island of summer isle from The Wicker Man (1973 film), which is fictitious.

how much do all the people in the world weight?

strange question; how much do all the people in the world weight? i was just wondering, of course you can't known exactly but average person size times 6 billions, would that work? how many tonnes?

Well the population is thought to be around 6,680,000,000 and of that number, about 2,200,000,000 are children, so I'd grossly average the weight of all at 110 pounds. If both numbers are anywhere near the mark, then it would be 734,800,000,000 pounds/333,299,676,779 kilograms/333,299,676 tonnes.--Fuhghettaboutit (talk) 11:46, 1 August 2008 (UTC)[reply]
Or 333,299,673 tonnes of living flesh.--Shantavira|feed me 11:55, 1 August 2008 (UTC)[reply]
wow, thank you. :) somehow i though it would be more, but i hadn't considered children.
What about the remaining 3 tonnes, Shantavira? Hair? Bowel obstructions? Chris Farley? Matt Deres (talk) 13:11, 1 August 2008 (UTC)[reply]
Well, according to World population, it was estimated that there were just over 6.684 billion of us last month. This gives "333 499 254 tonnes" using the 110 pound estimate above and google. Zain Ebrahim (talk) 13:28, 1 August 2008 (UTC)[reply]
To help UK readers understand the answer, this would be about 52,517,142,840 stone. Edison (talk) 14:52, 1 August 2008 (UTC)[reply]
For comparision, that is about 5 times as heavy as the Three Gorges Dam. Gandalf61 (talk) 15:06, 1 August 2008 (UTC)[reply]
Or about twice as heavy as <insert fat joke target here>. --98.217.8.46 (talk) 15:43, 1 August 2008 (UTC)[reply]
Don't even think about it.--The Fat Man Who Never Came Back (talk) 01:36, 3 August 2008 (UTC)[reply]
I think that those doing such calculations would do well to consider significant figures when stating their answers. (Not that it really matters for this purpose, but saying that they would weigh 333,299,676,779 kilograms, versus 333,299,676,780 kilograms or even 333,300,000,000 kilograms gives a false sense of confidence.) -- 128.104.112.147 (talk) 15:45, 1 August 2008 (UTC)[reply]
OK, make it 52,517,142,840 stone +/-25%. (I think all the contributors realized the humor of all the digits coming from a wild assed guess as to the 110 pounds. At least I hope so!)Edison (talk) 20:32, 1 August 2008 (UTC)[reply]
Re significant digits, I've lost about 1.2 pounds during July ^_^ hydnjo talk 02:24, 3 August 2008 (UTC)[reply]

Weather Prediction

I recall reading/hearing that you can predict the weather with around a 70% accuracy by simply stating that the weather tomorrow will be the same as the weather today - and that the billions spent on weather-predicting equipment only improves that margin to about 75-80% but that that % change is easily enough to make the investment worthwhile. Is there any truth to this or is it just basically gibberish? 194.221.133.226 (talk) 11:29, 1 August 2008 (UTC)[reply]

i would say that is true for most days, but every so often there will be one of those days when it's unpredictable and changes without warning, and than is when the weather predictors are more accurate because they monitor pressure and bars and isomorphs. for much of the week where i am its been the same each day, but there will come a time when that prediction will fails when it starts raining and then my predictions will be wrong for just one days untill the next changes. Mashpotatorman (talk) 11:38, 1 August 2008 (UTC)[reply]
(ec) How do you define "accuracy", and boil it down to a single number? It's probably easiest to measure temperature accuracy, but precipitation (amount and type) is probably more important in terms of financial and human losses. As an example, suppose that on Monday, I forecast a big, 12-inch snowstorm on Thursday. It turns out that the storm comes on Friday. In one sense, my forecast was lousy (off by 12 inches of snow on both Thursday and Friday). But, really, it was basically correct--people would know that a big snowstorm was coming later in the week, and plan appropriately (food at home, plow crews ready). That is, relatively small errors in the timing of a system can look like big errors in temperature or precipitation. I would also point out that having a day or two of advance notice for hurricanes (from satellites) and 10 minutes' notice for tornadoes (from radar) is quite valuable in terms of safety-of-life, but these advances will likely have a minimal impact on any "accuracy" statistic, because they are infrequent. -- Coneslayer (talk) 11:41, 1 August 2008 (UTC)[reply]
I was thinking in terms of the generic sunny/overcast/rainy/snowing/sunshine & showers rather than in-depth report. I.e. if it is sunny today then it's around 70% likely to be sunny tomorrow. I appreciate that is boiling the weather down to a child-like level of detail. 194.221.133.226 (talk) 13:33, 1 August 2008 (UTC)[reply]
Well, right. That's not what the "billions spent on weather-predicting" is aimed at, though. Weather prediction is not just about that little picture of a sun in your newspaper—it has all sorts of economic ramifications. --98.217.8.46 (talk) 13:50, 1 August 2008 (UTC)[reply]
Indeed. Arguably, predicting the weather tomorrow is less important than predicting the weather in an hour, or in 6 months. FiggyBee (talk) 13:55, 1 August 2008 (UTC)[reply]
That depends, of course, on whether or not you're expecting a hurricane, hailstorm, tornado, or icestorm tomorrow.... TenOfAllTrades(talk) 16:10, 1 August 2008 (UTC)[reply]

Just need a tiny bit of clarification of something that seemed vague to say the least?

I may simply be out of the loop; however, I looked up a wonderful man and his mission: Father Frank Provone and Prists for Life and I go this:

Priests for Life (PFL) is a Roman Catholic pro-life organization based in New York. It functions as a network to promote and coordinate pro-life activism with the primary strategic goal of ending abortion and euthanasia and to spread the Gospel of Life according to the encyclical of the same name written by Pope John Paul II.

I just don't see where a previous name is mentioned when "according to the encyclical of the same name " is said? Who is "the same name"?

Thanks!  :) —Preceding unsigned comment added by 205.144.73.92 (talk) 13:44, 1 August 2008 (UTC)[reply]

The "encyclical of the same name" is the Gospel of Life. FiggyBee (talk) 13:52, 1 August 2008 (UTC)[reply]

Keeping items unopened, a morality/legal/blahblah issue

Note: This is not asking for legal advice. I've been thinking about this for awhile, and I just remembered again while answering a question on the Entertainment board. Is it unlawful to buy new copies of games/dvd/music and not opening them and download copies of the same things instead?

For example, I buy DS games but still use the ROMs because 1) I don't have to carry multiple cartridges with my DS and don't risk damaging or losing them, 2) loads and saves faster (if you play dungeon crawlers like Etrian Odyssey, you'd know why you have to save pretty frequently, and it takes two ticks to save with a ROM and like 10 for the cart), and 3) it keeps my games in pristine condition. I do that with anime too, where I watch fansubbed or DVD rips and I have a big shelf of unopened anime.

So the question is... is this illegal (etc) to any extent that I keep my stuff new, and does it change whether I plan on selling them in the future? This is probably more of a question about morals, but I don't care too much about the moral response, since for the most part, Copyright Jesus probably doesn't see it as too evil, if at all. The jurisdiction can be anywhere, since I'd be interested if there's laws in other countries about it too, even though I'm in the US. --Wirbelwindヴィルヴェルヴィント (talk) 15:47, 1 August 2008 (UTC)[reply]

I'm not a lawyer but know something about UK copyright law. It seems to me that you have bought a piece of software which essentially is a licence to use it for your own purposes as prescribed by the copyright owners' terms and conditions, whereas, you have also downloaded/intend to dowload a similar piece of software without buying the necessary end-users' licence agreement, from a source that itself will, in all probability, not have paid for such a licence, nor a licence to act as an authorised distributor of said software. In my opinion, to quote my judicial friends on the bench, whilst the software you are keeping wrapped up allows you to use it perfectly legally, the download/ed copy does not. But I feel sure other more knowledgeable Wikipedians than I will offer further illumination. 92.22.118.51 (talk) 19:01, 1 August 2008 (UTC)[reply]
The answer to the question will almost certainly depend on your jurisdiction; as well, in many places there may not be a conclusive body of law (case or written) to unambiguously answer your query. The rules around what might constitute legitimate format shifting or space shifting of digital media have been the subject of a number of court cases in the United States (our space shifting article touches on a few). If you are in the United States, the DMCA may also come into play. TenOfAllTrades(talk) 19:49, 1 August 2008 (UTC)[reply]
In the UK, I believe you are allowed to copy such things for backup purposes, you can then use the backup the same as you can use the original, however you cannot use them both at the same time, or give away or sell one and keep the other. I don't know if downloading the backup would be allowed, you may have to make it yourself. This is just from memory, though, so may be completely wrong. --Tango (talk) 20:37, 1 August 2008 (UTC)[reply]
I'm pretty sure the law exists in the States to make backups yourself, and you're not allowed to sell the backup or something similar, but I have no idea about downloading the backup either. But to clarify my original question, I don't mean to download a backup while owning the physical copy and then selling the physical copy. My actual practice is more like never opening the physical copy and 99% of the time not ever planning on selling it (unless for some weird reason, I have two copies of the same thing, but then I'd still own at least a copy of the physical copy. Then I would download the identical item and use that copy. --Wirbelwindヴィルヴェルヴィント (talk) 22:01, 1 August 2008 (UTC)[reply]

I am looking for the titles, and if possible the location to watch these online for free, see www.watch-movies.net. In one, he is an advertising executive, but goes insane and is institutionalized, and then gets the other nutters to help him in his profession. Making adverts that are very true, and funny. The second him and his wife are trying to get pregnant and the doctor informs them that the reason they are having touble concieving is that his sperm is too hot and he then need to wear underwaer that has pockets of blocks of ice. Very funny. Any ideas people? Thanks —Preceding unsigned comment added by 193.115.175.247 (talk) 15:52, 1 August 2008 (UTC)[reply]

The first movie you're looking for is Crazy People. I don't know the second one. Note that it is very unlikely that any free online presentation of either film will have the permission of the copyright holders; it would be violation of our policy on external links to point to a site where you could find copyright violations. TenOfAllTrades(talk) 16:06, 1 August 2008 (UTC)[reply]
One more thought—our article on Dudley Moore includes a list of his films: Dudley Moore#Filmography. We have articles on many of those movies; have you looked through that list to see if you could find the film you seek? TenOfAllTrades(talk) 16:12, 1 August 2008 (UTC)[reply]

Hidden Pages

I just noticed that some Wikipedians have hidden pages that are supposed to be found out. Any idea how they look like? Can they be anything, pictures as well as text? Do they usually look like ordinary links?? 117.194.228.11 (talk) 16:44, 1 August 2008 (UTC)[reply]

Can you give an example? If people hide pages in their userspace they can be anything they want them to be. They can link to them or not. (A way to find them if they don't link to them is to look at the contributions of the user in question.) --98.217.8.46 (talk) 16:45, 1 August 2008 (UTC)[reply]
A better way to find stuff hidden in a user's userspace is Special:Prefixindex. Algebraist 16:56, 1 August 2008 (UTC)[reply]
No, that's cheating! But yes, sometimes a user makes a page in their userspace with no links to it. There are even a few Barnstar for those who find them, like this and this. 20I.170.20 (talk) 17:02, 1 August 2008 (UTC)[reply]
As this is a non-standard feature I'd guess they could be anything at all, even this:... But watch out for traps & guardians!87.102.86.73 (talk) 18:28, 1 August 2008 (UTC)
There was one user, I'll never remember who, so don't ask, who linked to their secret page via a period at the end of a sentence on their userpage. Useight (talk) 21:41, 1 August 2008 (UTC)[reply]

New Wikipedia.org organization

I just noticed that, today, the page www.wikipedia.org shows a different order of languages, with Spanish being second instead of German. Are they ordered now from more to less readers? What is the new criterion? --Taraborn (talk) 17:25, 1 August 2008 (UTC)[reply]

This issue was decided on Meta; see Top Ten Wikipedias poll. Sam Blacketer (talk) 18:36, 1 August 2008 (UTC)[reply]
Thanks! --62.57.212.165 (talk) 21:35, 1 August 2008 (UTC)[reply]

Problems of genealogy

I'll do an example: I'm a great-grandson of Alan. Sophia is a double-great-granddaughter of Alan: her parents are first cousins and Alan is grandfather of both.

My question is: Is Sophia my second cousin or my double second cousin??

I've this problem for Victor Amadeus II of Sardinia and his wife Anne Marie of Orleans Please, answer me. Thaaaaaaaaanks --84.222.154.178 (talk) 17:28, 1 August 2008 (UTC)[reply]

Double first cousin says that a double first cousin is where a pair of siblings reproduce with another pair of siblings (eg. My brother marries my sister-in-law), and a double second cousin is where double first cousins have children. It doesn't sound like that's what happening in this case, so they wouldn't be double second cousins, just regular second cousins. However, if I'm doing this right, they would share the same amount of DNA as first cousins once removed. --Tango (talk) 23:01, 1 August 2008 (UTC)[reply]

Thanks, Tango. --84.222.155.253 (talk) 09:43, 2 August 2008 (UTC)[reply]

Harbridge

Good evening, am very sorry to be a nuisance, it was only curiosity that made me write this, and the usual saying, ( you never ask , you never know } my name is Ron Harbridge i live in the north west, a place called astly..and was very intrigued to find out that there was a place called Harbridge even a village and a parish church witht he name Harbridge, i was hoping that you may posssibly have a contact telephone number of the Vicar of Harbridge Parish church, My wife wanted to drive down..until she realised where it was..So...if it is possible to pass on the telephone number i would be very greatful.my home e-mail is (removed for your protection). and many thanks in anticipation, Regards Ron Harbridge —Preceding unsigned comment added by 92.234.223.183 (talk) 17:34, 1 August 2008 (UTC)[reply]

Do you mean Astley Village in England? And where is Harbridge? − Twas Now ( talkcontribse-mail ) 17:50, 1 August 2008 (UTC)[reply]
There's a Harbridge in Hampshire, but we don't have an article about it. Deor (talk) 19:22, 1 August 2008 (UTC)[reply]
We do now :) - albeit a very stubby one (Harbridge). I'd say that it's likely it would be covered by either the Bournemouth or Salisbury area phone book, which hopefully you should be able to find either online or at your nearest large library. Grutness...wha? 00:36, 2 August 2008 (UTC)[reply]
I've had a quick web-search, and there don't appear to be contact details for All Saints' Church, Harbridge online anywhere - but it's likely you could find that information from the office of the local diocese - all the contact details for that office are at their site. Grutness...wha? 00:49, 2 August 2008 (UTC)[reply]

RIM BlackBerry!

Is it possible to get a BlackBerry Phone without the BlackBerry service? like ATT provides an offer without a Data Plan, but then there is a BlackBerry service too if I'm not wrong!The service that enables us to use the special BlackBerry button on the Phone!Don't we need to pay for that service too to make it possible to use the phone? Or is it not required??? —Preceding unsigned comment added by 123.252.224.172 (talk) 20:48, 1 August 2008 (UTC)[reply]

Yes, you can buy the phone and not get the service. You can use it as an ordinary cellphone without data access. -- Mwalcoff (talk) 01:32, 3 August 2008 (UTC)[reply]

Looking for an image

I am working on a class project that examines how, as a culture, we give lip service to fighting against anorexia but continue to sexualize it anyway. One of the examples I want to use is this image, in which the model does indeed look decidedly unhealthy, but for some reason is posed like a pin-up girl. To emphasize the comparison, I'd like to find an image where a happy, obviously meant-to-be-attractive woman is posed as similarly as possible to the anorexic woman. Does anyone know offhand where I could find an image like that? (The Betty Grable shot isn't bad, but I bet there's something a lot closer out there.) --Masamage 21:28, 1 August 2008 (UTC)[reply]

Here's a possibility. Very low-res scan, but maybe you could track down the original at a library somewhere. -- BenRG (talk) 22:43, 1 August 2008 (UTC)[reply]
Here's a shot of Marilyn that's pretty close. Matt Deres (talk) 23:12, 1 August 2008 (UTC)[reply]
Ask a friend or classmate to pose for you and take the picture yourself. -Arch dude (talk) 23:42, 1 August 2008 (UTC)[reply]
Not strictly a pin-up, but you might want to consider Ingres's painting "La Grande Odalisque", which the anorexia poster seems to be based upon. Other classical/neoclassical paintings such as Goya's two "Maja" paintings (nude and clothed), Manet's "Olympia", and Titian's "Venus d'Urbino" are also worth considering. Grutness...wha? 01:00, 2 August 2008 (UTC)[reply]
Oh, interesting. That painting is almost exactly the same, so I'll use it. Thanks! --Masamage 22:48, 2 August 2008 (UTC)[reply]
At least the anorexic has the right number of vertebrae. :) --Sean 14:58, 4 August 2008 (UTC)[reply]

Technical revaluation of currency

Concerning the technical revaluation of currency, i.e. deciding to change the numeral value of the currency, meaning that the denominations of the currency change, as do the prices, by the same factor, meaning that in the end, the same amount of currency buys the same amount of goods anyway, only the numbers are different. Can a country just suddenly decide to do this, or do they have to consult other countries first? JIP | Talk 21:38, 1 August 2008 (UTC)[reply]

Countries do it occasionally, often after run-away inflation. According to denomination (currency) the term for this is "redenomination" or "cutting zeroes." Zimbabwe is doing it now (or has done it recently), because transactions in trillions or quadrillions of dollars are ridiculous. Off the top of my head similar conversions happened in Yugoslavia and Turkey. The human psyche does not cope well with huge numbers with long trails of zeroes. It's harder to tell apart and work with large units like 10,000,000 and 100,000,000 than it is for the units 10 and 100. See Denomination (currency)#Redenomination for a better explanation. For book-keeping reasons, everyone needs to be on board with the change, so switch-overs that take place are well publicized. I don't know if another country can actually stop another from revaluing its currency, but I imagine communication takes place. 71.77.4.75 (talk) 22:53, 1 August 2008 (UTC)[reply]
I dug up this possibly relevant document which I just scanned over which seems to have something to say about the UN and financial reporting and promoting good practices. I am really too lazy to read it in depth but this would imply some measure of "mandatory-because-it's-politically-good-form" consulting goes on. The "mandatoriness" of the consulting is probably directly related to how important a particular country's currency is to the global economy. 71.77.4.75 (talk) 23:22, 1 August 2008 (UTC)[reply]
Many countries have revalued their currency in this way - Argentina is one that springs to mind, and IIRC some African countries (Ghana, perhaps?) have also done so, and several countries had to introduce new standards in this way during the hyperinflation of the Great Depression of 1929 - most famously Germany, but also Russia and several other countries. China did similar after the 1949 civil war. In several of these cases the name of the currency was slightly amended "Mark" to "Reichsmark" or "Yuan" to "Gold Yuan", or similar. Another famous example is France's revaluation of the Franc to 100 old Francs in the 1950s. I think anon 71....'s comments about consultation being good form are pretty accurate. I don't know of any compulsory need, but it makes sense to consult first. Grutness...wha? 00:28, 2 August 2008 (UTC)[reply]
The German inflation was in the early '20s, not the great depression. Hyperinflation suggests this applies to Russia and the several others also. Algebraist 12:00, 2 August 2008 (UTC)[reply]
And Germany did not just chop zeroes and rename the currency; they also changed the way it was backed, so this is not an example. See Rentenmark. --Anonymous, 19:09 UTC, August 2, 2008.
Yes, it previous cases, chopping off zeroes has been done one the hyperinflation has been dealt with (or, as the final stage in a plan to deal with it). It makes no difference economically how many zeros there are, so it's really just tidying up after yourself. However, previous cases happened before finances became extremely computerised - Zimbabwe has had to chop them off early because the computers couldn't cope, that wasn't an issue in the 20's! --Tango (talk) 02:29, 3 August 2008 (UTC)[reply]

Monasticism

How old does a man have to be in order to become a monk in the Roman Catholic Church? --Think Fast (talk) 21:53, 1 August 2008 (UTC)[reply]

[16] has some useful information. --Tango (talk) 22:48, 1 August 2008 (UTC)[reply]
The Catholic Encyclopedia under lemma Novice states that no minimum age is fixed by canon law for admission. If under the age the applicant requires consent of his parents / guardians. The article seems to imply that 14 to 15 is a minimum age. --Cookatoo.ergo.ZooM (talk) 11:33, 2 August 2008 (UTC)[reply]
To "become a monk," you join a religious order, which involves adhering to the community's standards, including its Rule. So it's not "the church" so much as the individual order. In addition, the answer depends on what you mean by "become a monk." You can join an order (e.g., as a postulant) without being under any vows; most orders will also require a period of temporary vows (e.g., vows for a fixed period, such as one or two years). Untimately Ultimately there's the full profession, the taking of perpetual vows. (The monastic vows are poverty, chastity, and obedience; the Benedictines add stability, which vow does not appear on the [stability] disambiguation page. OtherDave (talk) 04:24, 4 August 2008 (UTC)[reply]
I remember when I was at Buckfast Abbey School, one of the monks I got to know had been sent to the abbey at age 11 by his mother. I can't remember when he actually became a "fully fledged" monk, but he was certainly at that level by the time he started to look after the bees. I was quite surprised to find a Wiki article about him! Karl_Kehrle --Worm | mroW 12:36, 4 August 2008 (UTC)[reply]

August 2

Umbrella insurance

Does personal umbrella insurance typically cover discrimination lawsuits? —Preceding unsigned comment added by 70.177.10.216 (talk) 03:01, 2 August 2008 (UTC)[reply]

Genealogical Relationship

Can someone tell me the exact consanguinity relationship of Umberto II of Italy and his wife Maria José of Belgium. Thanks. --84.222.155.253 (talk) 09:42, 2 August 2008 (UTC)[reply]

For European royals, the two were fairly distantly related. In short, Umberto's father's mother's mother's mother's mother's parents (or his great-great-great-great-grandparents) Landgravine Amalie of Hesse-Darmstadt and Charles Louis, Hereditary Prince of Baden (who were themselves first cousins) were also the parents of Marie José's father's mother's mother's father (or Marie José's great-great-great-grandparents). Marco polo (talk) 21:24, 2 August 2008 (UTC)[reply]
That makes them 4th cousins once removed, by my calculation. Had Landgravine Amalie and Charles Louis been unrelated, Marie Jose and Umberto would have been 6th cousins once removed. -- JackofOz (talk) 21:39, 2 August 2008 (UTC)[reply]

Thank you very much!!!!!! But weren't they related with the family of the Two Sicilies, Marco Polo??? --84.222.154.142 (talk) 08:01, 4 August 2008 (UTC)[reply]

Family photo

Does someone knows the names of this people in this photo, from left to right. I need the exact names of Roberto I of Parma' children, if it's possible. Thanks. --84.222.155.253 (talk) 09:46, 2 August 2008 (UTC)[reply]

You forgot to attach the photo. -- JackofOz (talk) 09:49, 2 August 2008 (UTC)[reply]

Yes the photo is this —Preceding unsigned comment added by 84.223.192.213 (talk) 09:58, 2 August 2008 (UTC)[reply]

Our article has a list of all Roberto's children. Doesn't help put names to faces though. Algebraist 11:54, 2 August 2008 (UTC)[reply]

Budget for village fete

As an ex Marketing Director I have been asked to prepare a marketing plan for our 2009 village fete. I am not used to small budgets and will appreciate advice from those who are. Our annual fete features a big musical production featuring lead singers with the cast made up of 60 villagers. Attendance is 2,400 over 4 shows. I want to add programmes, souvenir booklets, medieval hats for the girls and heroes’ swords for the boys...plus any ideas that may be suggested. Trouble is I don’t know quantities to budget. Can anybody help from experience, please ?86.219.161.122 (talk) 10:41, 2 August 2008 (UTC)petitmichel[reply]

I suggest you figure it out as follows:
1) Find the price of a sword you consider to be appropriate, then multiply by the number of swords needed (get a few extra in case some break).
2) Repeat this process for hats, programs, souvenir booklets, and anything else you can think of.
3) Add them all up. Then maybe double it for a margin of error/cost overruns.
Just an order of magnitude estimate gives me something around $50,000. BTW, are you giving all this away or selling it ? If selling it, you can expect to get your money back, of course, but still need to have the money up front. StuRat (talk) 14:09, 2 August 2008 (UTC)[reply]

Thanks for the thought... but it only repeats the question. What I need is some idea of a typical demand. Everything hangs on that. From around 3,000 people how many are likely to buy and what? That is why I asked for help from experience.86.219.161.122 (talk) 15:09, 2 August 2008 (UTC)petitmichel[reply]

Er: your estimate of turnout has just risen from 2400 to 3000, an increase of some 25%. If you order enough for the lower figure and even only 30% of your first estimate buys your offerings and you calculate your costs on that 30% figure, you will break even. But, if you sell any more than that lower figure, you are in surplus. Of course, as an ex-Marketing Director, I am sure you will have hordes of willing and able amateurs around you who have experience of previous years' take up levels. I too used to be the Head of Administration in a large public organisation and always planned my programmes on domestic principles - and they always exceeded expectations. 92.20.198.103 (talk) 16:41, 2 August 2008 (UTC)[reply]
(ec) Maybe obvious, but would it not be the simplest solution to check what happened in 2008 and before? Forecasting a demand would be tricky, not knowing the specific economic data of the residents, the age / gender mix, the enthusiasm of the folks and so on. Extrapolating from a previous demand (if at all possible) gives, at least, a ballpark figure. --Cookatoo.ergo.ZooM (talk) 16:43, 2 August 2008 (UTC)[reply]
That was my first thought as well—though if this is the first year for swords and hats, he may have to make a wild guess. Best bet is still to talk to other people who have organized similar events, of course. Also worthwhile is to consider what you'll do with leftover hats and swords. (Can they be returned? Sold at a later event? Given to unarmed orphans?)
Depending on the budget you have available, you may also be able to consult with or hire a professional event planner who will be able to advise you on suppliers and quantities. TenOfAllTrades(talk) 17:16, 2 August 2008 (UTC)[reply]
Sort of random, but if the event takes place at night and there are lots of elementary to middle school aged kids and their parents (who have money) in attendance, glow-necklaces like these are very popular and have sold out at night events I've been too. I suppose the average per kid will probably about one. You might have the girls build their own hats using construction paper + sequins + glue + other accessories at a booth manned by volunteers. It will be messy, but it is probably less expensive overall then buying pre-fabricated hats. Also consider that children and swords will guarantee that sword fights are going to break out, and people will be whacked. It would be wise to make sure the swords are constructed out of something soft like foam and not plastic. If you choose the foam route, you can get a large foam sheet, spray paint it silver, and cut it into long skinny rectangles that are length of the blade + hilt. At a booth the corners at one end will be lopped off to make the end a triangle and the kids could stick on a construction paper crossguard and decorate it. Also keep in mind that girls may want swords too. 71.77.4.75 (talk) 02:54, 3 August 2008 (UTC)[reply]
Definitely look to volunteers whenever possible to lower costs. You may also consider cutting costs by trying to get local businesses to sponsor part of the event in exchange for advertisements in the programs. If you need cheap labor doing something you can't find volunteers for like folding hats or the like, high schools or colleges may be a good source. You could try asking the administration if you can post an announcement or fliers on campus and borrow a classroom for an hour or two after school lets out. 71.77.4.75 (talk) 03:22, 3 August 2008 (UTC)[reply]

Sorry about the 2,400/3,000 figure. The first is the ticket buying people. The balance are those interested. Although there is a lot of useful stuff here, most of it is basic marketing theory. And a lot is off the subject. We don't have a previous year to refer to, this is the first time we have tried to add stuff to generate extra income. What I am asking for (and that is all I need) is for somebody who has run an event such as I describe to share his/her experiences.86.209.27.67 (talk) 09:50, 3 August 2008 (UTC)petitmichel[reply]

Insure against rain for a start!--Artjo (talk) 11:18, 3 August 2008 (UTC)[reply]

Flight time

Can someone give me a rough idea of the non-stop direct flight time between London and Puerto Rico. Thanks--Artjo (talk) 12:26, 2 August 2008 (UTC)[reply]

According to this search site there are no non-stops. One stop flights are about 14 - 16 hours. Of course a real travel agent will be more informative :-) -hydnjo talk 13:09, 2 August 2008 (UTC)[reply]
My brief search agrees with Hydnjo's answer - no direct flights. One deal suggested a 13h50m journey including a 1h50m layover in Miami. Astronaut (talk) 13:32, 2 August 2008 (UTC)[reply]
Perhaps they meant the theoretical direct flight time ? StuRat (talk) 13:58, 2 August 2008 (UTC)[reply]
StuRat is right, but the above are fine for my needs, thanks all.--Artjo (talk) 14:54, 2 August 2008 (UTC)[reply]
Maybe use a Flight Simulator to predict it when you fly a jumbo? *shrug* JoshHolloway 16:37, 2 August 2008 (UTC)[reply]

Geography Geology or Hydrology ?

Is there a term for the line of fastest flow down a river. Note : Not Thalweg which is just deepest.

many thanks —Preceding unsigned comment added by 122.108.32.35 (talk) 13:30, 2 August 2008 (UTC)[reply]

I found some references which seem to indicate that thalweg could also refer to the line of fastest flow. See the Wiktionary definition and the NOAA's NWS glossary. However, the article Thalweg mentions (without a ref): "The thalweg is almost always the line of fastest flow in any river." which would imply otherwise. Zain Ebrahim (talk) 14:05, 2 August 2008 (UTC)[reply]

WAR TROPHY

WHAT IS THE HISTORY OF KEEPING SILVER TROPHIES IN OFFICERS MESS? —Preceding unsigned comment added by Bsgusain (talkcontribs) 13:43, 2 August 2008 (UTC)[reply]

Where else would you display the awards and trophies your Regiment had accumulated through the ages ?86.219.161.122 (talk) 15:06, 2 August 2008 (UTC)DT[reply]

Need more speaker/amp advice

Hi again, all. The build-your-own-bass-stack project has completed "Phase I", and we have (or had, for a few minutes) two working cabinets, a 2x10 and a 1x15. The boy was proud of our work. They were used in a real gig for the first time this morning.

As is frequently the case with the high-school-level battle-of-the-band format, there is much sharing of equipment. Apparently -- we do not know all the facts yet -- one overenthusiastic and under-mature bassist simply turned all the knobs up all the way and started banging away. By the time my kid got back to the venue, the feedback from the previous band was "dude, your stuff doesn't work."

With that as background, I could use any advice and guidance the WP community is willing to offer. My role in the project was power tools and woodworking; I know next to nothing about the electronics side of this. I've read several articles returned by searching for "diagnose" and "blown speaker".

  • We will disassemble the cabinets, and inspect and test each driver one at a time. But, what really are we looking for? The only visual thing we might be able to identify is torn cone -- is there anything else to look for?
  • How can we tell if damage has been done to the amplifier? What kind of damage is likely, and/or how would it manifest itself? We don't have a spare amp or backup cabinets.
  • I think I've learned that "amp too big for speakers" damages the speakers, but "amp not big enough for speakers" damages the amp somehow. If that's not really correct, could someone clarify?

Encyclopedia-grade responses not required. Thanks to all who can help us here. --Danh, 63.231.153.235 (talk) 20:08, 2 August 2008 (UTC)[reply]

Oh dear! I think it is more likely that the loudspeakers have been damaged. Cones dont tear like that (I dont think)-- you have to put your bot into them to tear them. To test each speaker cab on its own, obtain a 1.5v cell (from your torch, Tv remote etc) and apply across the leads to the speaker (cabinet). You should hear a thump and maybe see the cone move a little. If you dont and it doesn't, you have a blown voice coil. If so, its a new speaker (or you may be able to send back to the manufacturer for repair).
Instrument Amplifiers usually have protection circuits to allow them to be driven without damage into any load (including a short circuit) so its unlikely the amp has been damaged. —Preceding unsigned comment added by 79.76.199.184 (talk) 01:28, 3 August 2008 (UTC)[reply]

UPDATE: Things are not as dire as they first seemed. It now appears more likely that only one speaker has sustained real damage -- it buzzes like a herd of honeybees, which still has to be dealt with, but it's not like the entire project just got tossed in the crapper. Another source has pointed out, and our observations confirm, that there was stuff stacked on the amplifier, which has an cooling fan on the top pointing upward, which was thus blocked, which may have caused the amp to power-limit or turn itself off completely.

Can they really do that? That's a heckuva feature, if true! --Danh, 63.231.153.235 (talk) 03:45, 3 August 2008 (UTC)[reply]

If you're referring to "be driven without damage into any load", then yes. With the caveat that "be driven" doesn't mean "actually work", just that if the load is unsuitable the amp will "go into protection" and shut down temporarily. I'm not guaranteeing that all amps of any budget and vintage can do this :-) 81.187.153.189 (talk) 11:13, 3 August 2008 (UTC)[reply]

August 3

Beaches on north shore of Lake Erie

OK, it's not Maui, but are there any decent beaches on Lake Erie in Elgin, Norfolk or Haldimand county, Ontario? -- Mwalcoff (talk) 02:28, 3 August 2008 (UTC)[reply]

Norfolk has lots of beaches. Port Dover, Turkey Point, Long Point. In Elgin there is Port Stanley. Here is a list from Haldimand County. A little further east in Niagara, there are beaches in Fort Erie. I don't really know if any of these beaches are "decent", but Lake Erie is pretty cold and dirty so I wouldn't want to swim in it. Adam Bishop (talk) 15:43, 3 August 2008 (UTC)[reply]
It warms up this time of the summer moreso than the other great lakes because it's shallower, but I would also suggest staying out of it. I'm on the south side and we have mostly cliffs but some nice beaches and people go swimming without normally getting sick. Most people make sure they take a shower immediately after returning home. We have signs posted warning of contamination levels after the storms kick stuff up and the beaches with lifeguards don't let anyone in the waters then. I heard that people who test water quality think it's a pretty bad idea to ever go in there. -LambaJan (talk) 13:17, 4 August 2008 (UTC)[reply]
Are lakes Ontario and Huron as dirty? -- Mwalcoff (talk) 16:23, 4 August 2008 (UTC)[reply]

Traveling on a students budget

Is it possible to travel in the Middle East for an average $50/day? I’m thinking walking/bicycling, youth hostels, and living on staples here. . . Sħukrân --S.dedalus (talk) 03:33, 3 August 2008 (UTC)[reply]

Possibly, but travel to and from the Middle East will likely blow your budget. StuRat (talk) 03:47, 3 August 2008 (UTC)[reply]
Yeah, but there are ways of limiting the cost. For instance if a flight is overbooked you can agree to wait and take the compensation. Let’s just focus on cost on the ground to make it simple. --S.dedalus (talk) 04:20, 3 August 2008 (UTC)[reply]
It might also depend on where in "the Middle East" you are specifically referring. $50US will go a lot further in some countries there than others. --98.217.8.46 (talk) 13:10, 3 August 2008 (UTC)[reply]
That’s what I’d like to know. :) Where will $50 go far? Anybody know the answer? --S.dedalus (talk) 05:22, 4 August 2008 (UTC)[reply]
The prices in Jordan 10 years ago (I know, WAY out of date for your purposes, but perhaps a starting point?) were extraordinarily cheap, at least as far as basics like food and gasoline were concerned. I recall our tour guide treating the whole minibus to ice cream--we protested at the expense, and he then explained that ice cream cones cost something along the lines of 3 or 4 to the US$. This was a while back, and I'm sure my fuzzy memory can be outdone by folks with more recent experience. User:Jwrosenzweig editing as 71.112.34.57 (talk) 09:59, 4 August 2008 (UTC)[reply]
I was recently looking this up a little bit. I think Syria is probably much cheaper than Jordan. At this point Lebanon is probably closer to the Syria end of the spectrum. For that price range I'd stay away from the Emirates but if you want to go to the peninsula you might want to check out Oman or Yemen. I can't guarantee they'll be cheap but they'll most likely be a lot cheaper than the touristy places. -LambaJan (talk) 13:21, 4 August 2008 (UTC)[reply]
One warning, those places in the Middle East which are cheap are likely to also be dangerous. There are several reasons:
1) They were dangerous first, which made tourists avoid the place, which made the owners of hotels, etc., lower their costs to bring people in, despite the danger.
2) The hotel owners, etc., don't take in enough money to afford security guards, walls of sandbags, etc.
3) The region is poverty-stricken, which both brings about low prices and terrorism. StuRat (talk) 13:57, 4 August 2008 (UTC)[reply]
Thanks! Lebanon is on my short list, so I’m glad to hear it’s on the cheaper side of the spectrum.
StuRat, yeah, I’m aware of the potential dangers of visiting some parts of the Middle East. I also know about the countries you don’t want to mix on your passport, etc. However I’m young and have some experience and I figure it’s not going to get any easier for me to do these things later in life. It’s a risk I’m willing to take. --S.dedalus (talk) 19:17, 4 August 2008 (UTC)[reply]
OK, but be sure you take precautions and don't lose your head. StuRat (talk) 23:00, 4 August 2008 (UTC)[reply]

Longest building?

The list of largest buildings in the world states that the longest building in the world is some airport at 1.7 km. Excuse me, but what about Prora? Sure, it ain't used for anything, but it is, undoubtably, a building. Is there some criterion I'm missing or is the article simply incorrect? 83.188.196.191 (talk) 07:44, 3 August 2008 (UTC)[reply]

I don't know anything about Prora, but a glance at the article says that it's actually eight buildings, not one, and that that length is all eight of them together. --Masamage 08:37, 3 August 2008 (UTC)[reply]
And a glance at the satellite view linked from the article doesn't show any single building 1.7 km long. Franamax (talk) 08:55, 3 August 2008 (UTC)[reply]

On Line Chatting

So does it seem like people you meet chatting on line have more serious problems and bad events happen in their lives than average? Why is that? AndreaTrue77 (talk) 14:46, 3 August 2008 (UTC)[reply]

I'd chalk it up to the fact that you are likely to meet more people online than in "real life", thus increasing the chance that one of them would have had a serious problem or bad event. (It's more likely that one person in a chat room of 1000 people would have had a car accident in the past month than it would for one of 10 people in the hair stylist's waiting room.) You then have the fact that people are more likely to talk about abnormal events than routine events (In that chat room you'll probably here from the one person who has had a car accident [and the one person who has won the lottery] rather than the 900 people who are just doing the day-to-day slog.) You then throw in the Von Restorff effect, which means you'll remember the few people who have bad stories while forgetting the 900 people who said "can't complain". Finally, people are subject to confirmation bias, where once you have formulated a theory ("people online have more serious problems"), additional evidence that confirms the view is given more weight than evidence which conflicts with it. (That is, if two people, one online and one in real life, come up to you with stories about their car crashes, you are likely to view the first as confirming your theory, while the second would likely be dismissed as a random fluke or outlier - a special case not mandating adjustment to your theory.) Finally, there might even be some truth to the theory. I can't say for certain, but it may be that people who have serious problems are more likely to seek out socialization and companionship than other people, and may feel more comfortable doing it online. -- 128.104.112.147 (talk) 19:11, 3 August 2008 (UTC)[reply]
Would many people go online to talk about what a nice day it's been today, or other mundane topics? That gets really boring really quickly (much more so than chatting face to face about the same topic), and if they can't think of anything more substantial to talk about, the other party says "Gotta go now, see you next time". So, to avoid that, people tend to bring up meatier topics. They could discuss climate change, or the state of world poverty, or the global obesity epidemic, or the life of Emile Zola, but they'd need to be chatting to someone with a similar interest for that to work. One thing that everyone can relate to is a personal human issue, drama or tragedy, so that becomes the common denominator. It doesn't necessarily mean that that issue is dominating the writer's life away from the computer. It is very tempting, though, for the other party in the chat to form such a view, because typically that's all they know about their interlocutor and they get defined (in the listener's mind) by that issue. But we're all far more than just one issue, and every bad thing that happens is matched by many good things. -- JackofOz (talk) 00:05, 4 August 2008 (UTC)[reply]

Why does YouTube partner accounts main video on profile pages load automatically?

Why does YouTube partner accounts main video on profile pages load automatically without the viewer's consent? I criticize this feature because it discriminates against non-partner accounts, wastes bandwidth on the profile page.. They get their videos mark as "red diagonal ellipse" when searching for videos. I want to start a blog about this. That why I call these accounts "elite". Note: I'm not debating controversial subjects but want simple answers only. Jet (talk) 18:03, 3 August 2008 (UTC)[reply]

Are you really asking why? Because the answer is pretty obvious. It's about getting partners more hits, which means more people want to be partners, which benefits YouTube. Since YouTube is a service paid for by advertising and so forth, I don't think one really has a huge amount of leeway to complain about their relatively unobtrusive revenue-generating attempts. They aren't exactly forcing you to use their site. --98.217.8.46 (talk) 23:09, 3 August 2008 (UTC)[reply]
Because Google is capitalist. I'll blog it myself once I get around to it, was going to write an article on YouTube anyway. Avnas Ishtaroth drop me a line 05:12, 4 August 2008 (UTC)[reply]

Why is eternity symbolized with a sideways 8?

Who thought of this? It's really very confusing.

"Professor Dinglefuddy, I thought eternity meant forever. How come it says here it's only 8 years?" --Hey, I'm Just Curious (talk) 19:02, 3 August 2008 (UTC)[reply]

It's actually the infinity symbol. As per the article, we're really not sure why the symbol looks similar to the a sideways 8. -- 128.104.112.147 (talk) 19:19, 3 August 2008 (UTC)[reply]
Ummm, perhaps because (a) the symbol has no discernible beginning and no end, and (b) were it to be oriented upright, it's likely to get mistaken for the numeral "8"? This is just a guess, nothing fit to add to the article. -- Deborahjay (talk) 20:19, 3 August 2008 (UTC)[reply]

Sorry, I meant "infinity". I often use the words interchangably. Hey, I'm Just Curious (talk) 20:04, 3 August 2008 (UTC)[reply]

My all time favorite explanation is that it is a sand timer on it's side; therefore it will take an infinite amount of time to empty. 20I.170.20 (talk) 20:40, 3 August 2008 (UTC)[reply]
I understood it to be symbolic of a moebius strip (which has one continuous - i.e. infinite - side in two dimensions). Steewi (talk) 03:00, 4 August 2008 (UTC)[reply]
Oops - I just read the article above. It's a popular but implausible origin. I stand corrected. Steewi (talk) 03:03, 4 August 2008 (UTC)[reply]
To me, it's a twisted ouroboros. Avnas Ishtaroth drop me a line 05:11, 4 August 2008 (UTC)[reply]

Samsung SGH G600

Does the Samsung SGH G600 supporty SDHC memory or is it only the regualar microSDs?

89.241.242.129 (talk) 21:26, 3 August 2008 (UTC)[reply]

Everything I've found so far suggest it only supports microSD cards. Note that MicroSD cards are about a quarter the size of an SDHC card, so unless the phone has a dedicated port for SDHC it will not be compatible. The phone's manual should list which cards it will take. 20I.170.20 (talk) 21:38, 3 August 2008 (UTC)[reply]

Thanks very much, but you can actually get microSDHC cards nowadays, or so the article says on the wiki. I'll go with getting a regular 2gb microSD, just to be on the safe side. 89.241.242.129 (talk) 21:48, 3 August 2008 (UTC)[reply]

Ah, I see. Well then it's a possibility, though I know from experience than using SDHC cards in older devices can cause problems. Have a look at SD and SDHC compatibility issues for more info. 20I.170.20 (talk) 21:55, 3 August 2008 (UTC)[reply]
And, we do have a computing desk which may draw more attention to your question. -hydnjo talk 00:43, 4 August 2008 (UTC)[reply]

August 4

Heatwave sets off Tropical storm

Is this worthy of Wikipedia ?:


__________________________________________________________________________________


2008 Southern Heatwave

A high pressure that has formed over the South Central US has Texas, Louisiana, Arkansas, Oklahoma, now Colorado, Mississippi has caused and will cause temperatures to climb to record breaking levels for the next several days. It has also caused the formation of a tropical storm that will hit Port O'Connor, Texas and/or the Port Lavaca area of Texas in the next few days. Several electrical companies have suspended all "disconnect orders"("You don't pay your bill, you get disconnected") for the duration of this emergency. The heat wave has also caused the formation of servere thunderstorms that have struck places such as Marshall, Texas and Shreveport, Louisiana. These storms had 60 mph winds and penny sized hail. The high pressure cell has formed in the indicated areas since the middle of July and is the cause of the hellishly high temperatures, the servere weather and the tropical storm currently located near New Orleans, Louisiana. A front that was comming southward was also blocked by this high pressure cell. Heat advisories and heat warnings continue to be posted. The Weather Service has recommended that people stay out of the sun, drink plenty of fluids(NOT alcoholic ones), get to a air conditioned place, such as a library, mall.

________________________________________________________

  • External links
  • Two of the electrical companies that suspended "Disconnect orders" for non payment of bills

________________________________________________________________

I'm new here. I have seen articles here (long time reader, first time contributor) about then current (like the 2006 heat wave for instance)heat wave articles on here. I am also in the area affected by the heat wave. I've even tried to place this on "Wiki News" but there may be a glitch.65.173.105.133 (talk) 01:30, 4 August 2008 (UTC)[reply]

_________________________________________________________________________________________

I think that heatwave → water evaporation → clouds → storms is a normal function of the weather. Local weather advisories are not really notable, though a 2008 Southern USA heatwave article might be if there is something particularly unusual about it (long lasting, or record highs, or multiple deaths from heatstroke, or ...) Astronaut (talk) 18:18, 4 August 2008 (UTC)[reply]
Latest reports indicate that this heat wave is expanding into all areas of the Southern US. See Weather.com right now.65.173.105.133 (talk) 20:58, 4 August 2008 (UTC)[reply]

A better place to go to ask about this would be Wikipedia:Articles for creation. The reference desk is really intended for finding out information about non-wikipedia subjects - WP:AfC is specificially intended for new users to propose new articles like this. Grutness...wha? 23:40, 4 August 2008 (UTC)[reply]

I don't understand the "changing username" section

My username is Slater79. Last night, I made a request that I change my username to Runt. The answer came in the Username Changing section with all this complicated gobildeegoo, with language that only an experienced computer person would or could understand. There was all this language. I don't understand that stuff! I'm basically computer illiterate. Then I went and looked at the section tonight, and any mention of my request was GONE! I am completely confused! Why do they exclude and alienate and turn off the common person by using these terms and stuff; why not use language that's used by a layman, which is what I am? Anyway, I don't understand what the heck is going on. Why was my request taken away? I just need a friendly response on this, not some robot language that's inconcievable and non-understandable. It's easy to get downright flustered when this stuff happens. Could someone who is reading this, a FRIENDLY PERSON, a HUMAN BEING who can relate to OTHER HUMAN BEINGS, please explain to me WHY my request (a simple request) disappeared??? And stuff like that???? What is going on, and stuff like that? I don't have a background in computers. I don't understand. Can someone who is versed in computer and specifically Wikipedia stuff please explain it to me in LAYMAN TERMS, terms that a COMMON HUMAN BEING can understand? I am not a genius, but I AM a bright person, and with all of the editing and contributing that I have helped Wikipedia with for so long that has improved articles, I feel that I should at least get a simple explanation that's not in gobbledeegook language from some robot, or whatever it is! And why did my request seem to disappear? Am I asking for too much to get a friendy response from a REAL HUMAN BEING???? If you are a friendly human being out there who can explain this stuff to me in layman terms, I would certainly greatly appreciate it! Thank you! Slater79 (talk) 03:57, 4 August 2008 (UTC)[reply]

As for the "gobbledeebook", I have no idea, but your request for a change username was moved to the bottom of WP:USURP. This is because the username you want already exists and that's the page for taking already existing usernames. Useight (talk) 04:05, 4 August 2008 (UTC)[reply]

Thanks for replying to my thing, Useight. I meant "gobbledeegook", which is American slang for describing unnececarrily difficult language that is confusing. Thanks for clearing that up for me. So "Runt" is already being used, eh? Well, that explains that. Slater79 (talk) 04:16, 4 August 2008 (UTC)[reply]

You can still take "Runt", you'll just have to wait a week or so for the usurpation to go through. Useight (talk) 05:38, 4 August 2008 (UTC)[reply]

As for the gobbledegook—the problem is that over time Wikipedia has developed, mostly spontaneously, an incredibly intricate bureaucracy with dozens and dozens of little official policies, unofficial policies, words-from-on-high, official philosophies, unofficial philosophies, etc. to coordinate in haphazard fashion the thousands and thousands of different decisions that must be made in order to keep thousands of people from all over the world in league on the same project. When you aren't well-acquainted with these terms or policies or even the general style, it looks like you have been suddenly dropped into a legal journal. It doesn't help that half of the Wikipedians (to just make up a number) seem to love to take the time to show that they really know the policy and point out every exception to any rule they're about to tell you. (Well, it's not so much that they "seem to love it"—it's how authority works around here. He/she who knows the relevant policies backwards and forwards can win basically any policy-related argument.)

The best way to approach this is to say to whomever you are talking to, calmly, "I don't really understand the policy you are mentioning—could you summarize it for me in rather simple terms, with an emphasis on what I need to do in order to get what I want, if that is possible?" If you get exasperated, then everything will fall apart. (In my experience as a one-time admin, the easiest way to get rid of trolls is to throw policy at them about trolls, then they explode and start cursing, then you throw policy at them about cursing, then they go off the rails, then you can ban them forever. I'm not at all saying you're a troll—I'm just pointing out that emotional responses are often used as excuses for considering someone officially "unreasonable", whether or not the response is really justified.) --98.217.8.46 (talk) 12:33, 4 August 2008 (UTC)[reply]

Yeah, I get it. Thanks for the info! Slater79 (talk) 01:32, 5 August 2008 (UTC)[reply]

Fashion

Who introducted the full-skirted silhouette dubbed the "New Look" in the late 1940's? Which material is not illegal in the United States? "Hamd" means? Which woman has never been an editor of Vogue magazine? Which shoemaker began his career as a jeans buyer? —Preceding unsigned comment added by 66.152.240.82 (talk) 11:30, 4 August 2008 (UTC)[reply]

1) No idea. 2) Many. Do you have a list that we should choose from? 3) In English? Nothing. Though you may be interested in "Hem". Also, "Hemd" is the German word for shirt. 4) Billions of women have never been an editor of anything. Again, did you have a list for us to choose from? 5) Again, no idea. Dismas|(talk) 13:06, 4 August 2008 (UTC)[reply]
2)Denim. 4) Her Majesty the Queen
AlmostReadytoFly (talk) 14:25, 4 August 2008 (UTC)[reply]

69th Street, Upper Darby, PA F. W. Woolworth;s Store

I am trying to find out when the F. W. Woolworth's Store in 69th Street, Upper Darby, PA was opened.-Ladycat53fl (talk) 12:43, 4 August 2008 (UTC)[reply]

How common is knowledge of basic shapes among adult, and college educated, Americans?

I know, goofy question, but here's the background and why I ask.

In a college class some years ago, a communication experiment required 2 people to sit back to back and describe a picture for the other to draw; the drawer could not ask questions. One thing I said to draw was a triangle; I'm pretty sure I said a "right triangle." When we were finished, she said that part had stumped her; I said I understood because she may not have recalled the term "right triangle" from Geometry. However, she said she didn't remember what a triangle was!

Now, I could understand if it was an abstract concept - I can see the average Joe on the street not recalling exactly what a noun or verb are, for instance. But, a triangle is a very basic shape, the kind we learn in Kindergarten - or before, on Sesame Street - when I say it, I would presume most get a mental picture, because it has been so ingrained. (And, before you ask, no, it was not early in the morning - I could *easily* see that being the problem, but it was an afternoon course.)

So, are there any studies that show that knoledge of common shapes (tires are circles, etc.)? I know she could have excused something like dyslexia but with shapes by saying it wasn't common knowledge, not wanting to just tell anyone she had it, but my main query is whether it is common knowledge.209.244.30.221 (talk) 12:45, 4 August 2008 (UTC)[reply]

Certainly the shape of a triangle is common knowledge, as is a circle. A square is a bit tricky, though, as many people seem to confuse a square and a rectangle. Probably only those who have had a geometry class know what a quadrilateral is. Quite a few people know a pentagon, hexagon, and octagon, but probably not many know any other n-gons. The differences between ovals, ellipses, and slot-shapes is also lost on most people. When we go the 3 dimensions, it gets even worse, with many not even knowing what a sphere or a cube is. There is also confusion between 2- and 3-dimensional shapes, with "round" being used to mean a variety of shapes, including circles and spheres. StuRat (talk) 13:46, 4 August 2008 (UTC)[reply]
I should also note that visualization of 3D objects is one of the areas where men actually have a measurable advantage. StuRat (talk) 13:49, 4 August 2008 (UTC)[reply]
I once asked my four-year-old brother what shape a stop sign is and he told me "Octagon?". I was quite surprised. So, I'd say it varies greatly as to whether a person is going to know what trapezoids or parallelograms are. Useight (talk) 16:07, 4 August 2008 (UTC)[reply]

how do we know which house suits us.

my name is kaveri and my husband name is svs markendyulu his dob is 28th jan 1970 and mine is aug 3rd 1974. we saw one west facing house and we did not weather it is good for us and at present we are leaving in south facing housing is that good.

thanks —Preceding unsigned comment added by 76.205.200.220 (talk) 15:19, 4 August 2008 (UTC)[reply]

The relevance of your birthdays to house preference is highly subjective, and the house facing only slightly less so. Regarding facing, you'll likely want to consider where the sun will be relative to the house at given times of day. For instance, do you like bright sunlight when you wake up? Then east-facing windows in your bedroom are good. You may also want to consider the effects of shade on house temperature. Note, however, that these (and many other) house position criteria are independent of which direction the house as a whole faces. — Lomn 17:22, 4 August 2008 (UTC)[reply]
A south facing garden is often desirable as it will receive a high amount of sunlight throughout the day, as oppose to a north facing garden which will be in the shadow of the house. However, during summer it may become too hot to use at all. 20I.170.20 (talk) 17:34, 4 August 2008 (UTC)[reply]
That's in the northern hemisphere, of course. In the southern hemisphere it is the other way round. :) FiggyBee (talk) 19:38, 4 August 2008 (UTC)[reply]

British Patriotic songs

Does anzbody know any British patriotics songs that arent subject to copyright? I wish to use one on a youtube video without getting fined! Thanks v. much! --217.227.90.214 (talk) 18:34, 4 August 2008 (UTC)[reply]

Do you mean songs, or do you mean recordings? A great many well-known tunes (Rule, Britannia!, The British Grenadiers) are hundreds of years old and thus not protected by copyright. But individual recordings are protected in their own right - to find one that's out of copyright, you'll have to find a recording that was first published, publically performed or broadcast before 1958 (WP:IANAL and all that). FiggyBee (talk) 19:35, 4 August 2008 (UTC)[reply]
Others are God Save The Queen, Land of Hope and Glory, Jerusalem (although that's only for England - would you like patriotic songs for Wales, Scotland and Northern Ireland too, or are you only interested in those that are patriotic towards Britain?). You could use software to play an electronic instrumental version of one of these and record it, or look at the articles here for free recordings. 79.66.32.107 (talk) 22:17, 4 August 2008 (UTC)[reply]
Just found this template giving some patriotic songs. Might be useful. 79.66.32.107 (talk) 22:26, 4 August 2008 (UTC)[reply]

Relationship and facial desfiguration

You are in a relationship with someone and someday a dog bites his or her face and there is no plastic surgery to repair the damage. What would you do: leave the person and admit that your relationship were only with his or her face ? —Preceding unsigned comment added by Mr.K. (talkcontribs) 19:51, 4 August 2008 (UTC)[reply]

Surely that depends on whether "your relationship were only with his or her face", or not? FiggyBee (talk) 19:56, 4 August 2008 (UTC)[reply]

I think it can be assumed that there is a relationship of some depth, not just a relationship of the most superficial variety. With that in mind, I feel that there is a type of inertia that tends to keep relationships going despite a setback of the nature of a facial disfigurement. —Preceding unsigned comment added by Trom503 (talkcontribs) 20:05, 4 August 2008 (UTC)[reply]

Some people might be too distressed by the horrible scarring to be able to deal with it, even if they really did care about the other person as a person. Some people just have different levels of emotional capacity / squeamishness, and being capable of physical attraction to your partner is important. Also, they might have PTSD. (To answer the question for myself: even if my husband got chewed by a dog, I would stay with him.) --Masamage 20:19, 4 August 2008 (UTC)[reply]

Trite I know, but physical beauty really is only skin deep - and very temporary - the true and lasting beauty of a person lies in their inner core, their soul, their heart, their wit, their sincerity, their character, oh, I could go on and on. And if the dog in question had scarred my ex-wife - I would have married the dog. 92.1.150.57 (talk) 21:30, 4 August 2008 (UTC)[reply]

This sounds like an opening for discussion such as might be found in a forum. This is not a forum, but a reference desk. There are many forums on the internet where you can have long, involved conversations about this; please help keep this reference desk reference-desky (and not-deleted) by avoiding asking and answering these sort of questions. 79.66.32.107 (talk) 22:30, 4 August 2008 (UTC)[reply]

Baseball terminology

baseball terminology - what is a two seamer? —Preceding unsigned comment added by 216.215.18.70 (talk) 21:39, 4 August 2008 (UTC)[reply]


Its a type fastball Two-seam fastball --Nick910 (talk) 22:28, 4 August 2008 (UTC)[reply]

Born Under Punches

</obscurereference>

So, while I was out helping my mother with chores, my younger brother spotted a black car across the street. A bearded man came out and took pictures of our house, according to him. Being a paranoid family, my mother called police, only to find that "taking pictures isn't illegal".

Is this the FBI investigating me for whatever reason? Google Maps? A cat burglar? People at the bank taking pictures? Yeesh. I have crackpot theories, most of which do not make sense...

Any clue on what the person's motive was? Mr. Raptor (talk) 02:05, 5 August 2008 (UTC)[reply]

Is your house interesting looking? TastyCakes (talk) 02:19, 5 August 2008 (UTC)[reply]
Except for a stone wall. And we just moved in. My mother's boyfriend says it's Real Estate taking pictures to show comparisons of neighborhoods. Mr. Raptor (talk) 02:24, 5 August 2008 (UTC)[reply]